Haematology Flashcards

1
Q
  1. Anaemia
A Iron deficiency anaemia
B Beta-Thalassaemia
C Anaemia of chronic disease
D Blood loss
E Alcohol
F Vitamin B12 deficiency 
G Renal failure
H Aplastic anaemia
I Lead poisoning

1 A 35-year-old man presents to his GP with a 1-month history of increased tiredness. The patient also admits to diarrhoea and minor abdominal pain during this period. His blood tests reveal the following:

Hb 9.5 (13–18g/dL)
MCV 64 (76–96fL)
Fe 12.2 (14–31μmol/L) 
TIBC 74 (45–66μmol/L) 
Ferritin 9.2 (12–200μg/L)

Hb (haemoglobin); MCV (mean cell volume); Fe (iron); TIBC (total iron-binding capacity)

A

1) A

Iron deficiency anaemia (IDA; A) causes a hypochromic (pallor of the red blood cells on blood film due to reduced Hb synthesis), microcytic (small size) anaemia (low haemoglobin). A reduction in serum iron can be caused by a number of factors, including inadequate intake, malabsorption (coeliac disease; most likely cause in this case given diarrhoea and abdominal pain), increased demand (pregnancy) and increased losses (bleeding and parasitic infections). Further studies are required to distinguish IDA from other causes of microcytic anaemia: serum ferritin will be low, while total iron binding capacity (TIBC) and transferrin will be high.

Blood loss (D) will result in a normocytic anaemia as a consequence of a reduced number of circulating red blood cells. Common causes include gastrointestinal blood loss, heavy menstrual bleeding and certain surgical procedures.

Chronic alcohol (E) consumption directly causes a non-megaloblastic macrocytic anaemia. A poor diet in such patients also leads to folate and vitamin B12 deficiency which exacerbates the anaemia.

Chronic renal failure (G) is caused by the reduced production of red blood cells due to diminished secretion of erythropoietin by the damaged kidneys. This results in a normocytic, normochromic anaemia.

Lead poisoning (I) causes dysfunctional haem synthesis resulting in a microcytic anaemia. Lead poisoning leads to basophilic stippling, reflecting RNA found in red blood cells due to defective erythropoiesis.

How well did you know this?
1
Not at all
2
3
4
5
Perfectly
2
Q
  1. Anaemia
A Iron deficiency anaemia
B Beta-Thalassaemia
C Anaemia of chronic disease
D Blood loss
E Alcohol
F Vitamin B12 deficiency 
G Renal failure
H Aplastic anaemia
I Lead poisoning

2 A 56-year-old vagrant man presents to the accident and emergency department with weakness in his legs. The patient has a history of poorly controlled Crohn’s disease. His blood tests demonstrate Hb 9.4 (13–18 g/dL) and MCV 121 (76–96 fL). A blood film reveals the presence of hypersegmented neutrophils.

A

2) F

The majority of cases of vitamin B12 deficiency (F) occur secondary to malabsorption: reduced intrinsic factor production due to pernicious anaemia or post-gastrectomy, as well as disease of the terminal ileum. Clinical features will be similar to those of anaemia in mild cases, progressing to neuropsychiatric symptoms and subacute degeneration of the spinal cord (SDSC) in severe cases. Vitamin B12 deficiency results in a macrocytic megaloblastic anaemia as a result of inhibited DNA synthesis (B12 is responsible for the production of thymidine). Hypersegmented neutrophils are pathognomonic of megaloblastic anaemia.

Blood loss (D) will result in a normocytic anaemia as a consequence of a reduced number of circulating red blood cells. Common causes include gastrointestinal blood loss, heavy menstrual bleeding and certain surgical procedures.

Chronic alcohol (E) consumption directly causes a non-megaloblastic macrocytic anaemia. A poor diet in such patients also leads to folate and vitamin B12 deficiency which exacerbates the anaemia.

Chronic renal failure (G) is caused by the reduced production of red blood cells due to diminished secretion of erythropoietin by the damaged kidneys. This results in a normocytic, normochromic anaemia.

Lead poisoning (I) causes dysfunctional haem synthesis resulting in a microcytic anaemia. Lead poisoning leads to basophilic stippling, reflecting RNA found in red blood cells due to defective erythropoiesis.

How well did you know this?
1
Not at all
2
3
4
5
Perfectly
3
Q
  1. Anaemia
A Iron deficiency anaemia
B Beta-Thalassaemia
C Anaemia of chronic disease
D Blood loss
E Alcohol
F Vitamin B12 deficiency 
G Renal failure
H Aplastic anaemia
I Lead poisoning

3 A 65-year-old man is referred to the haematology department by his GP after initially presenting with tiredness, palpitations, petechiae and recent pneumonia. His blood tests reveal Hb 9.8 (13–18 g/dL), MCV 128 (76–96 fL), reticulocyte count 18 (25–100 × 109/L), 1.2 (2–7.5 × 109/L) and platelet count 125 (150–400 × 109/L).

A

3) H

Aplastic anaemia (H) is caused by failure of the bone marrow resulting in a pancytopenia and hypocellular bone marrow. Eighty per cent of cases are idiopathic, although 10% are primary (dyskeratosis congenita and Fanconi anaemia) and 10 per cent are secondary (viruses, SLE, drugs and radiation). The pathological process involves CD8+/ HLA-DR+ T cell destruction of bone marrow resulting in fatty changes. Investigations will reveal reduced Hb, reticulocytes, neutrophils, platelets and bone marrow cellularity as well as a raised MCV. Macrocytosis results from the release of fetal haemoglobin in an attempt to compensate for reduced red cell production.

Blood loss (D) will result in a normocytic anaemia as a consequence of a reduced number of circulating red blood cells. Common causes include gastrointestinal blood loss, heavy menstrual bleeding and certain surgical procedures.

Chronic alcohol (E) consumption directly causes a non-megaloblastic macrocytic anaemia. A poor diet in such patients also leads to folate and vitamin B12 deficiency which exacerbates the anaemia.

Chronic renal failure (G) is caused by the reduced production of red blood cells due to diminished secretion of erythropoietin by the damaged kidneys. This results in a normocytic, normochromic anaemia.

Lead poisoning (I) causes dysfunctional haem synthesis resulting in a microcytic anaemia. Lead poisoning leads to basophilic stippling, reflecting RNA found in red blood cells due to defective erythropoiesis.

How well did you know this?
1
Not at all
2
3
4
5
Perfectly
4
Q
  1. Anaemia
A Iron deficiency anaemia
B Beta-Thalassaemia
C Anaemia of chronic disease
D Blood loss
E Alcohol
F Vitamin B12 deficiency 
G Renal failure
H Aplastic anaemia
I Lead poisoning

4 A 56-year-old woman presents to her GP with increased tiredness in the past few weeks. A past medical history of rheumatoid arthritis is noted. Her blood tests demonstrate the following:

Hb 8.6 (11.5–16g/dL) 
MCV 62 (76–96fL)
Fe 10.2 (11–30μmol/L) 
TIBC 38 (45–66μmol/L) 
Ferritin 220 (12–200μg/L)
A

4) C

Anaemia of chronic disease (ACD; C) occurs in states of chronic infection and inflammation, for example in tuberculosis (TB), rheumatoid arthritis, inflammatory bowel disease and malignant disease. ACD is mediated by IL-6 produced by macrophages which induces hepcidin production by the liver. Hepcidin has the effect of retaining iron in macrophages (reduced delivery to red blood cells for erythropoiesis) and reduces export from enterocytes (reduced plasma iron levels). Laboratory features of ACD include a microcytic hypochromic anaemia, rouleaux formation (increased plasma proteins), raised ferritin (acute phase protein) as well as reduced serum iron and TIBC.

Blood loss (D) will result in a normocytic anaemia as a consequence of a reduced number of circulating red blood cells. Common causes include gastrointestinal blood loss, heavy menstrual bleeding and certain surgical procedures.

Chronic alcohol (E) consumption directly causes a non-megaloblastic macrocytic anaemia. A poor diet in such patients also leads to folate and vitamin B12 deficiency which exacerbates the anaemia.

Chronic renal failure (G) is caused by the reduced production of red blood cells due to diminished secretion of erythropoietin by the damaged kidneys. This results in a normocytic, normochromic anaemia.

Lead poisoning (I) causes dysfunctional haem synthesis resulting in a microcytic anaemia. Lead poisoning leads to basophilic stippling, reflecting RNA found in red blood cells due to defective erythropoiesis.

How well did you know this?
1
Not at all
2
3
4
5
Perfectly
5
Q
  1. Anaemia
A Iron deficiency anaemia
B Beta-Thalassaemia
C Anaemia of chronic disease
D Blood loss
E Alcohol
F Vitamin B12 deficiency 
G Renal failure
H Aplastic anaemia
I Lead poisoning

5 A 12-year-old Mediterranean boy presents to his GP with increased tiredness over the past few weeks which is affecting his ability to concentrate at school. Examination is normal. Blood tests demonstrate the following:

Hb 9.5 (13–18g/dL)
MCV 69 (76–96fL)
Fe 18.2 (14–31μmol/L) 
TIBC 54 (45–66μmol/L) 
Ferritin 124 (12–200μg/L)
A

5) B

Beta-Thalassaemia (B) is a genetic disorder characterized by the reduced or absent production of Beta-chains of haemoglobin. Mutations affecting the Beta-globin genes on chromosome 11 lead to a spectrum of clinical features depending on the combinations of chains affected. Beta-Thalassaemia minor affects one Beta-globin chain and is usually asymptomatic, but may present with mild features of anaemia. Haematological tests reveal a microcytic anaemia but iron studies will be normal, differentiating from iron deficiency anaemia. Beta-Thalassaemia major occurs due to defects of both Beta-globin chains and results in severe anaemia requiring regular blood transfusions, as well as skull bossing and hepatosplenomegaly.

Blood loss (D) will result in a normocytic anaemia as a consequence of a reduced number of circulating red blood cells. Common causes include gastrointestinal blood loss, heavy menstrual bleeding and certain surgical procedures.

Chronic alcohol (E) consumption directly causes a non-megaloblastic macrocytic anaemia. A poor diet in such patients also leads to folate and vitamin B12 deficiency which exacerbates the anaemia.

Chronic renal failure (G) is caused by the reduced production of red blood cells due to diminished secretion of erythropoietin by the damaged kidneys. This results in a normocytic, normochromic anaemia.

Lead poisoning (I) causes dysfunctional haem synthesis resulting in a microcytic anaemia. Lead poisoning leads to basophilic stippling, reflecting RNA found in red blood cells due to defective erythropoiesis.

How well did you know this?
1
Not at all
2
3
4
5
Perfectly
6
Q
  1. Blood transfusion (1)

A 22-year-old motorcyclist is involved in a road traffic accident, and is transfused two units of blood. 4 hours later he develops acute shortness of breath and hypoxia, and despite attempts at ventilation deteriorates rapidly and goes into respiratory arrest. An autopsy shows evidence of massive pulmonary oedema with granulocyte aggregation within the pulmonary microvasculature.

The most likely diagnosis is:

A Anaphylaxis
B ABO incompatible blood transfusion
C Fluid overload
D Transfusion related acute lung injury
E Air embolism
A

D

Transfusion related acute lung injury (TRALI) (D) is rare but is one of the leading causes of transfusion related mortality. It can present with acute shortness of break and hypoxia, as in this case, typically within 6 hours of receiving the transfusion. The classic presentation to look out for is that of non-cardiogenic pulmonary oedema, i.e. pulmonary oedema that is not due to fluid overload.

The underlying mechanism is not fully understood, but it is thought to involve HLA antibodies in the blood donor reacting with corresponding HLA antigens on the patient’s white blood cells. This leads to the formation of aggregates of white blood cells which become stuck in small pulmonary capillaries. The release of proteolytic enzymes from neutrophils and toxic oxygen metabolites causes lung damage, and subsequent non-cardiogenic pulmonary oedema which can be fatal. Treatment is essentially supportive, and includes stopping the transfusion, giving
IV fluids and ventilation if needed. TRALI can occur with platelets and FFP, as well as with packed red cells as in this case. You might find it helpful to remember the mechanism by rearranging ‘TRALI’ to form the word ‘TRAIL’, and think of the blood donor leaving a ‘trail’ of antibodies in the recipient.

An anaphylactic reaction (A) can also present immediately following a blood transfusion, but look out for clues such as a rash, urticaria and a wheeze to point you towards this diagnosis.

ABO incompatible transfusions (B) present with symptoms and signs of acute intravascular haemolysis, such as restlessness, chest or loin pain, fever, vomiting, flushing, collapse and haemoglobinuria. Shortness of breath and acute hypoxia are less common with this, and the pathological description given here at autopsy is characteristic of TRALI. The use of the term ‘pulmonary oedema’ in the question may have misled you to think of fluid overload (C). Whilst fluid overload is much more common, this patient has only received two units of blood and fluid overload would be less likely to cause such a rapid deterioration. These patients might have pedal oedema and bilateral crepitations on examination, and can be treated with diuretics.

An air embolism (E) can rarely occur if air is introduced into the blood bag, and can present with circulatory collapse. Again, the findings on autopsy from this case would not correlate with this diagnosis.

How well did you know this?
1
Not at all
2
3
4
5
Perfectly
7
Q
  1. Anaemia (2)
    A 24-year-old unemployed man presents to his GP with a 4-week history of flu-like symptoms and a persistent dry cough. On examination he has a maculopapular rash. A blood film reveals a haemolytic anaemia, and he is positive for cold agglutinins. The most likely organism implicated is:
A Streptococcus pneumoniae
B Mycoplasma pneumoniae
C Legionella pneumophilia
D Chlamydophila psittaci
E Borrelia burgdorferi
A

B
Autoimmune haemolytic anaemia is a form of mainly extravascular haemolysis, which is mediated by autoantibodies.

It is classified into warm and cold autoimmune haemolytic anaemia, according to the optimal temp at which the antibodies bind to RBCs.

This activates the classical pathway in the complement system, resulting in haemolysis. Cold AIHA is mediated by IgM antibodies, and as the name suggests these antibodies bind optimally at lower temperatures (28-31°C), resulting in anaemia that is aggravated in cold conditions. In severe cases, patients may suffer from Raynaud’s or acrocyanosis (purplish discolouration of peripheries). Most cases are idiopathic, but there are some specific causes worth remembering, as ‘Cold LID’:

  • -> Lymphoproliferative disease, e.g. CLL, lymphomas
  • -> Infections – mycoplasma, as in this case (B), EBV
  • -> Do not know, i.e. idiopathic!

This patient has typical features of mycoplasma pneumonia including a protracted history of flu-like symptoms (such as myalgia, arthralgia, headache) and a non-productive cough. Tx: includes avoiding cold conditions, use of chlorambucil, and treating the underlying cause. The other infectious agents listed here do not typically cause a cold haemolytic anaemia.

Warm AIHA on the other hand is mostly IgG mediated, and these anti- bodies have maximal reactivity at body temperature of 37°C. These antibodies attach to the membrane of red blood cells, and the ‘Fc’ portion is recognized by splenic macrophages. These remove part of the RBC membrane, which leads to the formation of spherocytes. Secondary causes may again include lymphoproliferative disease, but also drugs such as penicillin and autoimmune diseases such as SLE.
Tx: steroids, immunogolobulins and possibly splenectomy.

How well did you know this?
1
Not at all
2
3
4
5
Perfectly
8
Q
  1. Haematology of systemic disease
A Temporal arteritis
B Renal cell carcinoma
C Colorectal cancer
D Rheumatoid arthritis
E Miliary tuberculosis
F Acute pancreatitis
G Schistosomiasis
H Sarcoidosis
I Epstein–Barr infection

1 A 56-year-old woman visits her GP for a regular check-up for a chronic condition she suffers from. On examination, she has signs of long-term steroid therapy. There is ulnar deviation at her metacarpophalangeal joints. Blood tests reveal a microcytic hypochromic anaemia, low iron and total iron binding capacity, but a raised ferritin level.

A

1) D

Rheumatoid arthritis (RA; D) is an inflammatory disease that mainly affects the small joints of the hands but systemic involvement can be a feature, manifesting in the lungs (fibrosis), heart (pericarditis) and eyes (scleritis). RA is a cause of anaemia of chronic disease (ACD), which is mediated by IL-6 produced by macrophages. IL-6 induces hepcidin production by the liver which has the effect of retaining iron in macrophages (reduced delivery to red blood cells for erythropoiesis) and decreases export from enterocytes (reduced plasma iron levels). Laboratory features of ACD include a microcytic hypochromic anaemia, rouleaux formation and raised ferritin (acute phase protein).

Colorectal cancer (C) may result in iron deficiency anaemia (IDA) secondary to bleeding. IDA will demonstrate a microcytic anaemia, reduced ferritin and iron count and raised total iron binding capacity.

Miliary tuberculosis (E) may cause infiltration of the bone marrow leading to a leuko-erythroblastic picture on blood film. Other causes of a leuko-erythroblastic film include myelofibrosis, leukaemia, lymphoma and non-haemopoietic cancers (for example, breast cancer).

Acute pancreatitis (F) can result in a neutrophilia as a result of tissue inflammation. Other causes of neutrophilia include ulcerative colitis and corticosteroids.

Epstein–Barr virus (EBV; I) results in a reactive lymphocytosis. Other causes include cytomegalovirus, toxoplasmosis, hepatitis, rubella and herpes virus infection. Autoimmune disorders and neoplasia can also be causative.

How well did you know this?
1
Not at all
2
3
4
5
Perfectly
9
Q
  1. Haematology of systemic disease
A Temporal arteritis
B Renal cell carcinoma
C Colorectal cancer
D Rheumatoid arthritis
E Miliary tuberculosis
F Acute pancreatitis
G Schistosomiasis
H Sarcoidosis
I Epstein–Barr infection

2 A 45-year-old man presents to accident and emergency with an excruciating headache. Blood tests show an erythrocyte sedimentation rate of 110 mm/hour.

A

2) A

Temporal arteritis (A) is a vasculitis most commonly affecting the medium and large arteries of the head. It is also known as giant cell arteritis due to the inflammatory cells that are visualized on biopsy. Prominent temporal arteries with regional tenderness, coupled with an erythrocyte sedimentation rate (ESR) of more than 60mm/hour is highly suggestive of temporal arteritis. ESR may be raised due to increase plasma proteins (fibrinogen, acute phase proteins or immunoglobulin) or due to reduced packing of red blood cells (anaemia). Other causes of a raised ESR include myeloma, polymyalgia rheumatica and autoimmune disease.

Colorectal cancer (C) may result in iron deficiency anaemia (IDA) secondary to bleeding. IDA will demonstrate a microcytic anaemia, reduced ferritin and iron count and raised total iron binding capacity.

Miliary tuberculosis (E) may cause infiltration of the bone marrow leading to a leuko-erythroblastic picture on blood film. Other causes of a leuko-erythroblastic film include myelofibrosis, leukaemia, lymphoma and non-haemopoietic cancers (for example, breast cancer).

Acute pancreatitis (F) can result in a neutrophilia as a result of tissue inflammation. Other causes of neutrophilia include ulcerative colitis and corticosteroids.

Epstein–Barr virus (EBV; I) results in a reactive lymphocytosis. Other causes include cytomegalovirus, toxoplasmosis, hepatitis, rubella and herpes virus infection. Autoimmune disorders and neoplasia can also be causative.

How well did you know this?
1
Not at all
2
3
4
5
Perfectly
10
Q
  1. Haematology of systemic disease
A Temporal arteritis
B Renal cell carcinoma
C Colorectal cancer
D Rheumatoid arthritis
E Miliary tuberculosis
F Acute pancreatitis
G Schistosomiasis
H Sarcoidosis
I Epstein–Barr infection

3 A 38-year-old man from Nigeria presents to his GP with progressive shortness of breath, cough and painful rashes on his lower legs. Blood tests reveal a monocytosis. Chest X-ray demonstrates bihilar lymphadenopathy.

A

3) H

Sarcoidosis (H) is a granulomatous disease characterized by the presence of non-caseating granulomas in multiple organs, most commonly affecting the lungs. Diagnosis of sarcoidosis is usually a matter of excluding other diseases but chest X-ray (bihilar lymphadenopathy), CT scanning and lung biopsy can all help. Blood tests commonly reveal a monocytosis; monocytes are contributory to the pathogenesis of granulomatous disease. Other causes of monocytosis include brucellosis, typhoid, vari- cella zoster infection and chronic myelo-monocytic leukaemia (CMML).

Colorectal cancer (C) may result in iron deficiency anaemia (IDA) secondary to bleeding. IDA will demonstrate a microcytic anaemia, reduced ferritin and iron count and raised total iron binding capacity.

Miliary tuberculosis (E) may cause infiltration of the bone marrow leading to a leuko-erythroblastic picture on blood film. Other causes of a leuko-erythroblastic film include myelofibrosis, leukaemia, lymphoma and non-haemopoietic cancers (for example, breast cancer).

Acute pancreatitis (F) can result in a neutrophilia as a result of tissue inflammation. Other causes of neutrophilia include ulcerative colitis and corticosteroids.

Epstein–Barr virus (EBV; I) results in a reactive lymphocytosis. Other causes include cytomegalovirus, toxoplasmosis, hepatitis, rubella and herpes virus infection. Autoimmune disorders and neoplasia can also be causative.

How well did you know this?
1
Not at all
2
3
4
5
Perfectly
11
Q
  1. Haematology of systemic disease
A Temporal arteritis
B Renal cell carcinoma
C Colorectal cancer
D Rheumatoid arthritis
E Miliary tuberculosis
F Acute pancreatitis
G Schistosomiasis
H Sarcoidosis
I Epstein–Barr infection

4 A 66-year-old presents to his GP with severe weight loss over 1 month as well as tiredness. Blood tests reveal an increased erythrocyte, haemoglobin and erythropoietin count.

A

4) B

Renal cell carcinoma (RCC; B) is the most common type of renal cancer. Secondary polycythaemia may be associated with RCC as a result of increased erythropoietin (EPO) production. Secondary polycythaemia can be distinguished from primary polycythaemia as in the former there is an increase in blood EPO levels, whereas in the latter EPO levels decrease. Other causes of secondary polycythaemia include chronic hypoxia (high altitude, smoking, lung disease, cyanotic heart disease), renal disease (cysts, renal artery stenosis, hydronephrosis) and solid tumours (renal cell carcinoma and hepatocellular carcinoma).

Colorectal cancer (C) may result in iron deficiency anaemia (IDA) secondary to bleeding. IDA will demonstrate a microcytic anaemia, reduced ferritin and iron count and raised total iron binding capacity.

Miliary tuberculosis (E) may cause infiltration of the bone marrow leading to a leuko-erythroblastic picture on blood film. Other causes of a leuko-erythroblastic film include myelofibrosis, leukaemia, lymphoma and non-haemopoietic cancers (for example, breast cancer).

Acute pancreatitis (F) can result in a neutrophilia as a result of tissue inflammation. Other causes of neutrophilia include ulcerative colitis and corticosteroids.

Epstein–Barr virus (EBV; I) results in a reactive lymphocytosis. Other causes include cytomegalovirus, toxoplasmosis, hepatitis, rubella and herpes virus infection. Autoimmune disorders and neoplasia can also be causative.

How well did you know this?
1
Not at all
2
3
4
5
Perfectly
12
Q
  1. Haematology of systemic disease
A Temporal arteritis
B Renal cell carcinoma
C Colorectal cancer
D Rheumatoid arthritis
E Miliary tuberculosis
F Acute pancreatitis
G Schistosomiasis
H Sarcoidosis
I Epstein–Barr infection

5 A 24-year-old man has recently returned from a trip to Kenya. He presents to his GP with abdominal pain, fever and on examination has hepatosplenomegaly. Blood tests reveal a marked eosinophilia.

A

5) G

Schistosomiasis (G) is a parasitic disease caused by Schistosoma spp. It is particularly common in Asia, Africa and South America. The risk of bladder cancer is increased in urinary forms of schistosomiasis. The immune response to parasitic infection involves eosinophils and hence a marked eosinophilia is characteristic. Other causes of eosinophilia besides parasitic infection include allergic disease (asthma, rheumatoid arthritis, polyarteritis), neoplasms (Hodgkin’s lymphoma, non-Hodgkin’s lymphoma) as well as certain drugs (NSAIDs).

Colorectal cancer (C) may result in iron deficiency anaemia (IDA) secondary to bleeding. IDA will demonstrate a microcytic anaemia, reduced ferritin and iron count and raised total iron binding capacity.

Miliary tuberculosis (E) may cause infiltration of the bone marrow leading to a leuko-erythroblastic picture on blood film. Other causes of a leuko-erythroblastic film include myelofibrosis, leukaemia, lymphoma and non-haemopoietic cancers (for example, breast cancer).

Acute pancreatitis (F) can result in a neutrophilia as a result of tissue inflammation. Other causes of neutrophilia include ulcerative colitis and corticosteroids.

Epstein–Barr virus (EBV; I) results in a reactive lymphocytosis. Other causes include cytomegalovirus, toxoplasmosis, hepatitis, rubella and herpes virus infection. Autoimmune disorders and neoplasia can also be causative.

How well did you know this?
1
Not at all
2
3
4
5
Perfectly
13
Q
  1. Anaemia (3)
    A 7-year-old boy is taken ill from school on a cold December day, with a presumed viral infection. On returning home that day, he beings to feel even more unwell with a very high fever, headache and abdominal pain. His father begins to worry that his skin has taken on a yellow tinge, and the boy says his urine is now a dark reddy-brown colour. He is taken to the GP and after several tests the presence of ‘Donath–Landsteiner antibodies’ is reported. This child is suffering from:
A Paroxysmal cold haemoglobinuria
B Paroxysmal nocturnal haemoglobinuria
C Sickle cell disease
D Acute intermittent porphyria
E Epstein–Barr virus
A

A
Paroxysmal cold haemoglobinuria (A) is a rare form of autoimmune haemolytic anaemia. It usually affects children in the acute setting after an infection, and the key in this case is the presence of sudden haemoglobinuria and jaundice after exposure to a cold temperatures. IgG autoantibodies usually form after an infection, and bind to red blood cell surface antigens, inducing variable degrees of intravascular haemolysis in the cold. The antibodies are known as ‘Donath–Landsteiner antibodies’.

Analysis of the urine will confirm the presence of haemaglobinuria, and blood tests often reveal a normocytic or macrocytic anaemia. It is possible to test indirectly for the IgG antiglobulins at a low temperature, as in this case. Blood transfusion may be required if the anaemia is severe, but in children who have an acute onset with an antecedent infection, it is usually a transient and self limiting condition.
Paroxysmal nocturnal haemoglobinuria (B) is another rare acquired dis- ease, but one that is potentially life threatening. The resulting defect in the red cell membrane leads to intravascular haemolysis. The disease has three aspects: the most common way for it to present is with a haemolytic anaemia, which may cause haemoglobinuria, especially overnight. The second aspect is thrombophilia, which can present with visceral thrombosis (e.g. CNS, pulmonary, mesenteric). The third aspect is deficient haematopoiesis which can cause a pancytopenia with aplastic anaemia. You can remember this as PNH = Pancytopenia – New thrombus – Haemolytic anaemia. The latest diagnostic test is flow cytometry, which can detect absent membrane proteins on red blood cells. This has largely replaced the ‘Ham’s test’ (which was used to show that a patient’s eryth- rocytes are lysed if the blood is acidified). Treatment is with thrombo- prophylaxis, and the monocolonal antibody eculizumab may have a role.

Sickle cell disease (C) can lead to vaso-occlusive crises precipitated by the cold, but this would present with severe pain due to microvascular occlusion and Donath Landsteiner antibodies would not be present.

Acute intermittent porphyria (D) is an autosomal dominant condition caused by deficiency of an enzyme involved in haem synthesis (porphobilinogen deaminase). This can lead to accumulation of toxic haem precursors, which cause neurovisceral symptoms. The urine can characteristically turn a deep red colour on standing.

Epstein–Barr virus (E) alone would not cause the symptoms described in this case, though it can trigger paroxysmal cold haemoglobinuria.

How well did you know this?
1
Not at all
2
3
4
5
Perfectly
14
Q
  1. Anaemia (4)

A 21-year-old student has recently been diagnosed with coeliac disease. She presents to her GP complaining of increased tiredness and shortness of breath on climbing stairs.

Which of the following are most likely to be raised in this patient?

A Serum iron
B Haematocrit
C Transferrin
D Ferritin
E Mean cell haemoglobin
A

C

This patient is suffering from iron deficiency anaemia, a common complication in coeliac disease. The tiredness and shortness of breath are common symptoms. Causes can include blood loss (e.g. upper or lower GI bleeding, menstruation), malabsorption (as in this case), dietary deficiency (rare in adults but can be seen in children) or infestation with parasitic worms (the most common cause worldwide). Blood tests characteristically reveal a low mean cell volume, mean cell haemoglobin (E) and mean cell haemoglobin concentration. A blood film may reveal hypochromic red blood cells with anisocytosis (variation in cell size) and poikilocytosis (variation in cell shape). The red blood cell distribu- tion width (RDW) (a measure of the variation of the width of red blood cells) may be increased initially.

Serum iron levels (A) can be measured directly, but are unreliable and may be increased if the patient has started on iron supplements, as the levels increase straight away. Levels of ferritin (D), the intracellular protein that stores iron, may also be low and this is the most sensitive test. However, it is also an acute phase protein, and so may be falsely elevated in the presence of inflammation or malignancy which coexists with the iron deficiency anaemia (therefore a normal ferritin level cannot exclude IDA). The haematocrit (B) is the percentage of red blood cells in the blood, and it may be reduced in IDA.

Transferrin (C) is a glycoprotein that binds to iron in the blood, and lev- els may be increased in IDA as the liver produces greater amounts (you can think of this as the liver trying to compensate for the little iron it has available, so it makes more of the binding carrier for iron). Total iron-binding capacity (TIBC) is a laboratory test used to give a measure of the capacity of the blood to bind iron with transferrin, and it too would be raised in IDA. The TIBC is reduced in anaemia of chronic disease, possibly because the body produces less transferrin to prevent the pathogens that require iron for metabolism obtaining it.
IDA can be treated with oral iron supplements, with an expected rise in the haemoglobin of 1g/dL per week. Do not forget that iron supplements characteristically lead to production of black stools.

How well did you know this?
1
Not at all
2
3
4
5
Perfectly
15
Q
  1. Anaemia (5)
    A 34-year-old woman with known Addison’s disease is brought to the GP by her husband, as he is concerned that she keeps falling over at night. On examination the GP notes that she has conjunctival pallor. A thorough neurological examination reveals absent knee jerks, absent ankle jerks and extensor plantars bilaterally. Which of the following is the most sensitive test for the condition she has developed?
A Anti-intrinsic factor antibodies
B Anti-endomysial cell antibodies
C Anti-smooth muscle antibodies
D Anti-parietal cell antibodies
E Anti-voltage gated calcium channel antibodies
A

D
This woman has developed pernicious anaemia leading to vitamin B12 deficiency. It can be associated with other autoimmune conditions, such as Addison’s disease or thyroid disease. Specifically, she has developed a condition called subacute combined degeneration of the cord (SACD) which has led to symmetrical loss of dorsal columns (resulting in loss of touch and proprioception leading to ataxia, and LMN signs) and cor- ticospinal tract loss (leading to UMN signs), with sparing of pain and temperature sensation (which is carried by spinothalamic tracts). The ataxia and loss of joint position sense have resulted in her falling at night, which may be exacerbated by optic atrophy – another manifestation of vitamin B12 deficiency.

Remember that vitamin B12 is found in meat, fish and dairy products. More common causes of vitamin B12 deficiency can be related to diet (e.g. vegans) or to malabsorption. It is absorbed in the terminal ileum after binding to intrinsic factor produced by the parietal cells in the stomach. Causes of malabsorption can therefore be related to the stomach (e.g. post gastrectomy, pernicious anaemia), or due to the terminal ileum (e.g. Crohn’s, resection of the terminal ileum, bacterial overgrowth).

Pernicious anaemia is caused by an autoimmune atrophic gastritis when autoantibodies are produced against parietal cells and intrinsic factor itself. The lack of B12 impairs DNA synthesis in red blood cells, leading to the production of large, megaloblastic erythrocytes.

Blood tests and a blood film may reveal several features that are worth remembering:

  • Low haemoglobin
  • High MCV
  • Low platelets and WCC if severe
  • Low serum B12
  • Hypersegmented neutrophils
  • Megaloblasts in the bone marrow
  • Cabot rings in RBCs (remnants of the nuclear membrane seen in pernicious anaemia, lead poisoining and other forms of megaloblastic anaemia)

Intrinsic factor antibodies (A) can be found in approximately 50% of patients, and are specific for pernicious anaemia but not as sensitive as anti-parietal cell antibodies (D) which are found in >90% of patients. However, anti-parietal cell antibodies can also be found in approximately 10% of normal people, and 40 per cent of people who have atrophic gastritis without pernicious anaemia. The Schilling test is no longer commonly used for diagnosis.

Anti-endomysial cell antibodies (B) are found in coeliac disease (with a specificity of approximately 95%), anti-smooth muscle cell anti- bodies (C) are found in autoimmune hepatitis and primary biliary cirrhosis, and anti-voltage gated calcium channel antibodies (E) are found in Lambert–Eaton syndrome (a variant of myasthenia gravis).

How well did you know this?
1
Not at all
2
3
4
5
Perfectly
16
Q
  1. Macrocytic anaemia

A 58-year-old woman is referred to a haematology clinic following repeated chest infections and epistaxis. On examination she has conjunctival pallor and some petechial rashes on her forearms, but no organomegaly. Her blood tests reveal a pancytopenia, and an MCV of 112. Her drug history includes omeprazole, carbamazepine, gliclazide, metformin, paracetamol, and simvastatin. BM biopsy reveals a hypocellular marrow. The most likely diagnosis is:

A Aplastic anaemia
B Myelodysplasia
C Hypothyroidism
D Chronic myeloid leukaemia
E Myeloma
A

A
Causes of macrocytosis can be divided into:
1 Megaloblastic, e.g. folate and B12 deficiency
2 Non-megalobastic, causes of which can be remembered as RALPH = reticulocytosis (e.g. in haemolysis), alcohol, liver disease, pregnancy and hypothyroidism)
3 Other haematological disorders, e.g. myelodysplasia, aplastic anaemia, myeloma, myeloproliferative disorders

This woman is suffering from aplastic anaemia (A), where the bone marrow stops producing cells leading to a pancytopenia. Bone marrow examination is needed to confirm the diagnosis, and shows a hypocellular bone marrow. Causes of aplastic anaemia can be primary or secondary. Primary causes can be congenital (e.g. Fanconi’s anaemia) or idi- opathic acquired aplastic anaemia. Secondary causes include drugs (all the Cs – cytotoxics, carbamazepine, chloramphenicol, anticonvulsants such as phenytoin), ionizing radiation and viruses (e.g. hepatitis, EBV). This woman’s aplastic anaemia is secondary to long-term carbamaz- epine therapy for hypothyroidism. Hypothyroidism (C) alone may lead to macrocytosis, but is not the underlying cause for her pancytopenia. Patients with aplastic anaemia can present with features of the pancyto- penia, such as recurrent infections (from a low white cell count), bleed- ing and petechial rashes (from a low platelet count) and features of anaemia.
Treatment of aplastic anaemia is with supportive therapy, such as red cell transfusions and platelets, allogenic bone marrow transport which can be curative, or immunosuppression with, for example, ciclosporin and antithymocyte globulin (ATG).

Myelodysplasia (B) is a group of disorders caused by ineffective haema- topoeisis, so results in pancytopenia with increased marrow cellularity. Chronic myeloid leukaemia (D) would typically result in a very high white blood cell count, and a hypercellular bone marrow as there is active pro- duction of cells. Myeloma (E) can also cause a macrocytic or normocytic anaemia, but a bone marrow biopsy would show increased plasma cells.

How well did you know this?
1
Not at all
2
3
4
5
Perfectly
17
Q
  1. Hepatomegaly
    A 50-year-old diabetic man sees his GP complaining of generalized tiredness and a painful right knee. He is found on examination to have 5 finger breadths of hepatomegaly. An X-ray of his right knee is reported as showing chondrocalcinosis. His blood tests are likely to reveal:
A Raised MCV
B Raised total iron binding capacity
C Reduced serum ferritin
D Reduced iron level
E Raised transferrin saturation
A

E
This man has hereditary haemachromatosis, an inherited disorder of iron metabolism. It is particularly common in those of Celtic descent, and the gene responsible for the majority of cases is the HFE gene on chromosome 6.
Increased iron absorption leads to deposition to multiple organs including:
- the liver (hepatomegaly, deranged LFTs)
- joints (arthralgia, chondrocalcinosis)
- pancreas (diabetes)
- heart (dilated cardiomyopathy)
- pituitary gland (hypogonadism and impotence)
- adrenals (adrenal insufficiency)
- skin (slate grey skin pigmentation)

Blood tests can show deranged LFTs as in this case, as well as a raised serum ferritin, raised serum iron, reduced or normal total iron binding capacity and raised transferrin saturation (E) (>80%).

Remember that the TIBC measures the blood’s capacity to bind iron with transferrin. The transferrin saturation is the ratio of serum iron to TIBC ×100, and represents the percentage of iron binding sites on transferrin that are occupied by iron. It is typically 20–40%, but is raised in haemachromatosis. This is because TIBC is usually low or normal whilst serum iron levels are high, so the percentage of transferrin occupied by iron is increased.

The following table summarizes the common laboratory findings for various conditions:

Liver biopsy with Perl’s staining or Prussian blue staining can demon- strate iron overload in haemachromatosis. This can be used to quantify iron loading and determine the severity of the disease. MRI is also a less invasive way to accurately gauge iron concentrations in the liver.

Treatment options include lifetime regular venesection to reduce iron levels, maintenance of a low iron diet, and treatment with iron chelators if venesection is not possible. Patients with haemachromatosis who have developed liver cirrhosis are at increased risk of developing hepatocellular carcinoma.

How well did you know this?
1
Not at all
2
3
4
5
Perfectly
18
Q
  1. Plasma cell disorders (1)
    A 64-year-old woman is seen in the haematology clinic with generalized bone pain and recurrent infections. Following a set of blood tests, a skeletal survey reveals multiple lytic lesions and a bone marrow biopsy reports the presence of >10% plasma cells. Her blood tests are most likely to have shown:

A Raised calcium, normal alkaline phosphatase, raised ESR
B Normal calcium, raised alkaline phosphatase, normal ESR
C Raised calcium, raised alkaline phosphatase, raised ESR
D Raised calcium, normal alkaline phosphatase, raised CRP
E Normal calcium, normal alkaline phosphatase, raised CRP

A

A
This woman has multiple myeloma, a cancer of plasma cells. The symptoms can be remembered using the mnemonic BRAIN:
- Bone pain (due to osteoclast activation leading to hypercalcaemia and the presence of lytic lesions on a skeletal survey, characteristically with a ‘pepperpot skull’ appearance),
- Renal failure (which can be secondary to one or a combi- nation of: hypercalcaemia, tubular damage from light chain secretion, or secondary amyloidosis),
- Anaemia (typically normocytic),
- Infections (particularly pneumonias and pyelonephritis), and
- Neurological symp- toms (such as a headache and visual changes from hyperviscosity, or confusion and weakness from the hypercalcaemia).

Diagnostic criteria for symptomatic myeloma:

  • > Clonal plasma cells >10% on bone marrow biopsy
  • > A paraprotein in the serum or urine – most commonly IgG
  • > Evidence of end-organ damage related to the plasma cell disorder (commonly referred to by the acronym ‘CRAB’):
    - Calcium – high
    - Renal insufficiency
    - Anaemia
    - Bone lesions (e.g. lytic lesions, or osteoporosis with compression factors)

Blood tests may reveal a high calcium but the alkaline phosphatase is often normal (A) (in contrast to other malignancies, with osteolytic metastases and raised alkaline phosphatase).

The bone disease in myeloma is thought to be mediated by over- expression of the ‘RANK ligand’ by bone marrow stroma, which activates osteoclasts. Peripheral blood films can reveal the presence of rouleaux formation (stacks of red blood cells which occur because high plasma protein concentrations make the cells stick to each other, which also causes the high ESR).

Beta 2 microglobulin levels (a component of MHC class 1 molecules) can also be measured, and are an important prognostic indicator. Along with albumin levels, the level of beta 2 microglobulin forms part of the International Staging System for myeloma.

Treatment of multiple myeloma includes high dose chemotherapy, with the possibility of stem cell transplantation in younger patients.

How well did you know this?
1
Not at all
2
3
4
5
Perfectly
19
Q
  1. Plasma cell disorders (2)
    A 67-year-old woman presented with polyuria and polydipsia on a background of ongoing bone pain. Her blood tests revealed a high calcium, and a serum electrophoresis was sent. Her serum paraprotein was 25g/L and a BM biopsy revealed 6% clonal plasma cells. The most likely diagnosis is:

A Plasma cell dyscrasia
B Monoclonal gammopathy of undetermined significance
C Smouldering myeloma
D Multiple myeloma
E Hypercalcaemia with no evidence of underlying malignancy

A

D
This question tests your understanding of the diagnostic criteria for plasma cell disorders. Do not forget that:
1 Symptomatic myeloma (D):
Clonal plasma cells on bone marrow biopsy
Paraprotein in either serum or urine
Evidence of end-organ damage attributed to the plasma cell
disorder, commonly remembered using the acronym ‘CRAB’ (Calcium – high, Renal insufficiency, Anaemia and Bone lesions)
2 Asymptomatic (smouldering) myeloma (C):
Serum paraprotein >30 g/L AND/OR
Clonal plasma cells >10 per cent on bone marrow biopsy AND NO myeloma-related organ or tissue impairment
3 Monoclonal gammopathy of undetermined significance (MGUS) (B): Serum paraprotein <30g/L AND
Clonal plasma cells <10 per cent on bone marrow biopsy AND NO myeloma-related organ or tissue impairment
This woman has multiple myeloma because she has evidence of end organ damage in the form of hypercalcaemia. You can automatically exclude asymptomatic myeloma and MGUS on this basis!
MGUS itself is usually asymptomatic and does not normally require treatment. Patients will undergo regular blood tests to check their para- protein levels because of the small risk of transformation to multiple myeloma (approximately 1–2 per cent per year).
Plasma cell dyscrasia (A) is a more general term for cancers of the plasma cells, of which MGUS is the most common. This term also encompasses multiple myeloma, solitary plasmacytoma of bone, extramedullary plasmacytoma, and Waldenström’s macroglobulinaemia amongst others. A plasmacytoma is a discrete neoplastic mass of plasma cells within the bone marrow or elsewhere (extra-medullary). There is no evidence of myeloma in this condition, but a serum paraprotein is sometimes present (usually IgM).
Waldenström’s macroglobulinaemia is a chronic, indolent disorder, also known as ‘lymphoplasmacytic lymphoma’. It is essentially a clonal dis- order of B cells, characterized by a high level of IgM. This leads to fea- tures of hyperviscosity and vascular complications. Because of the high IgM levels it used to be thought of as related to multiple myeloma, but is now classified as a lymphoproliferative disease (similar to a low grade non-Hodgkins lymphoma).

How well did you know this?
1
Not at all
2
3
4
5
Perfectly
20
Q
  1. Platelet count (1)
    A 39-year-old motorcyclist is admitted following a road traffic accident com- plicated by severe burns. Several days later he is due to go home, when ooz- ing is noted from his cannula site and he has several nose bleeds. Repeat blood tests reveal an Hb of 12.2g/dL, WCC of 11.2×109/L, and platelets of 28×109/L. A coagulation screen shows a prolonged APTT and PT. He also has a reduced fibrinogen and raised D-dimers. The most likely diagnosis is:
A Liver failure
B Disseminated intravascular coagulation
C Thrombotic thrombocytopenic purpura
D Aplastic anaemia
E Heparin induced thrombocytopenia
A

B
This man has developed disseminated intravascular coagulation (DIC) (B) following his severe burns. DIC is widespread pathological activation of the clotting cascade in response to various insults. The cascade is activated in various ways: one mechanism is the release of a transmembrane glycoprotein called ‘tissue factor’ in response to cytokines or vascular damage. This results in fibrin formation, which can eventually cause occlusion of small and medium sized vessels and lead to organ failure. At the same time, depletion of platelets and coagulation proteins can result in bleeding (as in this case).
It can be caused by a wide range of factors, which can be remembered using the mnemonic ‘I’M STONeD!’: Immunological (e.g. severe allergic reactions, haemolytic transfusion reactions), Miscellaneous (e.g. aortic aneurysm, liver disease), Sepsis, Trauma (including serious tissue injury, burns, extensive surgery), Obstetric (e.g. amniotic fluid embolism, placental abruption), Neoplastic (myeloproliferative disorders as well as solid tumours such as pancreatic cancer), and Drugs and toxins.
Patients with DIC can present with rapid onset of shock, widespread bleeding, bruising and renal failure, or more insidiously (for example in the cases of malignancy). Blood tests will typically reveal a thrombocytopenia, raised PT and APTT, decreased fibrinogen and increased
D dimers. D dimers are fibrinogen degradation products, which form from intense fibrinolytic activity. The blood film may show the presence of schistocytes (broken red blood cells). The following table may help you to differentiate between the blood test results for various conditions.

Liver failure (A) is less likely to cause a thrombocytopenia, reduced fibrinogen and raised D dimers, and the history of burns points more towards DIC.

TTP (C) would not typically result in raised D dimers either, and the PT and APTT are not normally prolonged. This condition usually has other clinical features too, including a fever and fluctuating CNS signs.

Aplastic anaemia (D) does not typically cause abnormalities in clotting, and heparin induced thrombocytopenia (HIT) (E) is most likely to present paradoxically with throm- bosis rather than bleeding.

How well did you know this?
1
Not at all
2
3
4
5
Perfectly
21
Q
  1. Platelet count (2)
    A 46yo woman is brought to A&E by her daughter, who reports that she had been feeling unwell for a few days with a fever and is now hallucinating.
    O/E: temp 38.9°C, pale, widespread purpura over both arms. Blood tests: Hb of 9.1g/dL, platelet count: 60×109/L, creatinine: 226 and urea: 16.7. Blood film shows shistocytes. Most likely diagnosis:

A Weil’s disease
B Glandular fever
C Idiopathic thrombocytopenic purpura (ITP)
D Thrombotic thombocytopenic purpura (TTP)
E Haemolytic uraemic syndrome (HUS)

A

D
This woman has TTP, rare but potentially fatal haematological emergency. It consists of 6 key features:

1 MAHA
2 A fever
3 Renal failure
4 Fluctuating CNS signs, e.g. seizures, hallucinations, hemiparesis, decreased consciousness
5 Haematuria/proteinuria
6 Low platelet count

You can remember these as ‘MARCH with low platelets’.

TTP typically affects adults and is thought to occur due to a deficiency of a protease that is responsible for cleaving multimers of von Willebrand factor. The resulting formation of large vWF multimers stimulates platelet aggregation and fibrin deposition in small vessels. This in turn causes microthrombi to form in blood vessels, impeding the blood supply to major organs such as the kidneys, heart and brain. Haemolysis occurs and shistocytes form because of the sheer stress on rbcs as they pass through the microscopic clots.
Urgent plasma exchange can be lifesaving in patients with TTP, so it is important to consider this diagnosis early in patients who have unexplained thrombocytopenia and anaemia. The mortality rate is reported as >95% if untreated.

Idiopathic thrombocytopenic purpura (C) is an autoimmune disorder caused by IgG antibodies against platelets in most cases. Treatment depends on the platelet count and the presence of bleeding, but includes steroids, anti-D, immunosuppressants and splenectomy.

Haemolytic uraemic syndrome (D) typically affects young children infected with a specific strain of E. coli called O157, which produces a verotoxin that attacks endothelial cells and results in MAHA. The anaemia, thrombocytopenia, renal failure and presence of shistocytes could be caused by HUS in this question, but it is less likely given the patient’s age, the presence of neurological symptoms and the absence of preceding symptoms of gastroenteritis.

The presence of a fever may have led you to consider an infectious cause such as Weil’s disease (A) or glandular fever (B). Weil’s disease is caused by the spirochaete Leptospira interrogans, and is spread by infected rat urine. Although it can cause an abrupt onset of renal failure and a fever, it would not typically result in thrombocytopenia or features of MAHA. Glandular fever is also unlikely in this scenario: whilst it can cause palatal petechiae, it does not typically present with hallucinations or purpura, and a thrombocytopenia and anaemia are again less likely.

How well did you know this?
1
Not at all
2
3
4
5
Perfectly
22
Q
2. Blood transfusion (2)
A 43-year-old woman is transfused three units of blood as an emergency fol- lowing prolonged haematemesis. A few minutes later she becomes restless, and complains of chest pain. On examination she is pyrexial and tachycardic with a blood pressure of 95/60. There is bleeding at the site where her cannula is inserted, and urinalysis reveals haemoglobinuria. The most likely diagnosis is:
A Anaphylaxis
B ABO incompatible blood transfusion
C Myocardial infarction
D Graft versus host disease
E Bacterial contamination
A

B
An ABO incompatible blood transfusion (B) can occur immediately after a transfusion has been given. For example, if group A, B or AB blood is given to a group O patient, the patient’s anti-A and anti-B antibodies attack the blood cells in the donor blood. The most severe form of reaction is thought to occur if group A red cells are transfused to a group O patient. Even just a few millilitres of blood can trigger a severe reaction within a few minutes. These reactions can also occur with platelets or fresh frozen plasma because they also contain anti-red cell antibodies.
Symptoms can include chills, fever, pain in the back, chest or along
the IV line, hypotension, dark urine (intravascular haemolysis), and uncontrolled bleeding due to DIC. In this case, the management involves stopping the transfusion immediately and taking blood samples for
FBC, biochemistry, coagulation, repeat x-match, blood cultures and direct antiglobulin test, and contacting the haematology doctor as soon as possible. The blood bank should also be urgently informed because another patient may have also been given incompatible blood. These patients require fluid resuscitation and possibly inotropic support. They should be transferred to ICU if possible.
These reactions can be prevented through measures such as proper iden- tification of the patient from sample collection through to administering the blood product and careful labelling of the samples. If the patient is unconscious, then careful monitoring of observations before, during and after the transfusion can help to detect signs of a reaction as early as possible.
An anaphylactic reaction (A) can also present immediately following a blood transfusion, but look out for clues such as a rash, urticaria and a wheeze to point you towards this diagnosis. A myocardial infarction (C) is less likely in this setting, and would not cause intra- vascular haemolysis. Graft versus host disease (D) is a rare form of a delayed transfusion reaction which can occur in immunosuppressed patients, where lymphocytes from donor blood can attack the host. This can result in liver failure, diarrhoea, skin rashes and bone mar- row failure. Bacterial contamination (E) would also cause a fever and may lead to hypotension and tachycardia, so can be difficult to dif- ferentiate from ABO incompatibility. However, these reactions would not typically cause pain or haemoglobinuria. Usually a very high fever, rigors, and profound hypotension can be clues to this diagnosis in the question.

How well did you know this?
1
Not at all
2
3
4
5
Perfectly
23
Q
  1. Haemolytic anaemia
A Hereditary sherocytosis
B Sickle cell anaemia
C Beta-Thalassaemia
D Glucose-6-phosphate
dehydrogenase deficiency
E Pyruvate kinase deficiency
F Autoimmune haemolytic anaemia
G Haemolytic disease of the newborn
H Paroxysmal nocturnal haemoglobinuria
I Microangiopathic haemolytic anaemia

1 A 48-year-old woman diagnosed with chronic lymphocytic leukaemia devel- ops jaundice and on examination is found to have conjunctival pallor. Direct antiglobulin test is found to be positive at 37°C.

A

1) F

Autoimmune haemolytic anaemia (AIHA; F) is caused by autoantibodies that bind to red blood cells (RBCs) leading to splenic destruction. AIHA can be classified as either ‘warm’ or ‘cold’ depending on the tempera- ture at which antibodies bind to RBCs. Warm AIHA is IgG mediated, which binds to RBCs at 37°C; causes include lymphoproliferative dis- orders, drugs (penicillin) and autoimmune diseases (SLE). Cold AIHA is IgM mediated which binds to RBCs at temperatures less than 4°C; this phenomenon usually occurs after an infection by mycoplasma or EBV. Direct antiglobulin test (DAT) is positive in AIHA and spherocytes are seen on blood film.

Hereditary spherocytosis (A) and hereditary eliptocytosis are both auto- somal dominant disorders that result in RBC membrane defects and extravascular haemolysis.

Beta-Thalassaemia (C) results in defects of the globin chains of haemoglobin. As a consequence, there is damage to RBC membranes causing haemolysis within the bone marrow.
Pyruvate kinase deficiency (E) is an autosomal recessive genetic disorder that causes reduced ATP production within RBCs and therefore reduces survival.

Paroxysmal nocturnal haemoglobinuria (H; PNH) is a rare stem cell disorder which results in intravascular haemolysis, haemoglobinuria (especially at night) and thrombophilia. Ham’s test is positive.

How well did you know this?
1
Not at all
2
3
4
5
Perfectly
24
Q
  1. Haemolytic anaemia
A Hereditary sherocytosis
B Sickle cell anaemia
C Beta-Thalassaemia
D Glucose-6-phosphate
dehydrogenase deficiency
E Pyruvate kinase deficiency
F Autoimmune haemolytic anaemia
G Haemolytic disease of the newborn
H Paroxysmal nocturnal haemoglobinuria
I Microangiopathic haemolytic anaemia

2 An 18-year-old man presents to accident and emergency after eating a meal containing Fava beans. He is evidently jaundiced and has signs sug- gestive of anaemia. The patient’s blood film reveals the presence of Heinz bodies

A

2) D

Glucose-6-phosphate dehydrogenase deficiency (G6PD deficiency; D) is caused by an X-linked recessive enzyme defect. G6PD is an essential enzyme in the red blood cell pentose phosphate pathway; the pathway maintains NADPH levels which in turn supply glutathione to neutralize free radicals that may otherwise cause oxidative damage. Therefore, G6PD deficient patients are at risk of oxidative crises which may be precipitated by certain drugs (primaquine, sulphonamides and aspirin), fava beans and henna. Attacks result in rapid anaemia, jaundice and a blood film will demonstrate the presence of bite cells and Heinz bodies.

Hereditary spherocytosis (A) and hereditary eliptocytosis are both auto- somal dominant disorders that result in RBC membrane defects and extravascular haemolysis.

Beta-Thalassaemia (C) results in defects of the globin chains of haemoglobin. As a consequence, there is damage to RBC membranes causing haemolysis within the bone marrow.
Pyruvate kinase deficiency (E) is an autosomal recessive genetic disorder that causes reduced ATP production within RBCs and therefore reduces survival.

Paroxysmal nocturnal haemoglobinuria (H; PNH) is a rare stem cell disorder which results in intravascular haemolysis, haemoglobinuria (especially at night) and thrombophilia. Ham’s test is positive.

How well did you know this?
1
Not at all
2
3
4
5
Perfectly
25
Q
  1. Haemolytic anaemia
A Hereditary sherocytosis
B Sickle cell anaemia
C Beta-Thalassaemia
D Glucose-6-phosphate
dehydrogenase deficiency
E Pyruvate kinase deficiency
F Autoimmune haemolytic anaemia
G Haemolytic disease of the newborn
H Paroxysmal nocturnal haemoglobinuria
I Microangiopathic haemolytic anaemia

3 A 10-year-old girl presents to accident and emergency with jaundice. Blood tests reveal uraemia and thrombocytopenia. A peripheral blood film demon- strates the presence of schistocytes.

A

3) I

Microangiopathic haemolytic anaemia (I) is caused by the mechanical destruction of RBCs in circulation. Causes include thrombotic thrombocytopenic pupura (TTP), haemolytic uraemic syndrome (HUS; E. coli O157:57), disseminated intravascular coagulation (DIC) and systemic lupus erythematosus (SLE). In all underlying causes, the potentiation of coagulation pathways creates a mesh which leads to the intravascular destruction of RBCs and produces schistocytes (helmet cells). Schistocytes are broken down in the spleen, raising bilirubin levels and initiating jaundice.

Hereditary spherocytosis (A) and hereditary eliptocytosis are both auto- somal dominant disorders that result in RBC membrane defects and extravascular haemolysis.

Beta-Thalassaemia (C) results in defects of the globin chains of haemoglobin. As a consequence, there is damage to RBC membranes causing haemolysis within the bone marrow.
Pyruvate kinase deficiency (E) is an autosomal recessive genetic disorder that causes reduced ATP production within RBCs and therefore reduces survival.

Paroxysmal nocturnal haemoglobinuria (H; PNH) is a rare stem cell disorder which results in intravascular haemolysis, haemoglobinuria (especially at night) and thrombophilia. Ham’s test is positive.

How well did you know this?
1
Not at all
2
3
4
5
Perfectly
26
Q
  1. Haemolytic anaemia
A Hereditary sherocytosis
B Sickle cell anaemia
C Beta-Thalassaemia
D Glucose-6-phosphate
dehydrogenase deficiency
E Pyruvate kinase deficiency
F Autoimmune haemolytic anaemia
G Haemolytic disease of the newborn
H Paroxysmal nocturnal haemoglobinuria
I Microangiopathic haemolytic anaemia

4 A 9-year-old boy from sub-Saharan Africa presents to accident and emergency with abdominal pain. On examination the child is found to have dactylitis. Blood haemoglobin is found to be 6.2 g/dL and electrophoresis reveals the diagnosis.

A

4) B

Sickle cell anaemia (B) is an autosomal recessive genetic haematological condition due to a point mutation in the Beta-globin chain of haemoglobin (chromosome 11); this mutation causes glumatic acid at position six to be substituted by valine. Homozygotes for the mutation (HbSS) have sickle cell anaemia while heterozygotes (HbAS) have sickle cell trait. The mutation results in reduced RBC elasticity; RBCs therefore assume a sickle shape which leads to the numerous complications associated with a crisis. Blood tests will reveal an anaemia, reticulocytosis and raised bilirubin. Haemoglobin electrophoresis will distinguish between HbSS and HbAS.

Hereditary spherocytosis (A) and hereditary eliptocytosis are both auto- somal dominant disorders that result in RBC membrane defects and extravascular haemolysis.

Beta-Thalassaemia (C) results in defects of the globin chains of haemoglobin. As a consequence, there is damage to RBC membranes causing haemolysis within the bone marrow.
Pyruvate kinase deficiency (E) is an autosomal recessive genetic disorder that causes reduced ATP production within RBCs and therefore reduces survival.

Paroxysmal nocturnal haemoglobinuria (H; PNH) is a rare stem cell disorder which results in intravascular haemolysis, haemoglobinuria (especially at night) and thrombophilia. Ham’s test is positive.

How well did you know this?
1
Not at all
2
3
4
5
Perfectly
27
Q
  1. Haemolytic anaemia
A Hereditary sherocytosis
B Sickle cell anaemia
C Beta-Thalassaemia
D Glucose-6-phosphate
dehydrogenase deficiency
E Pyruvate kinase deficiency
F Autoimmune haemolytic anaemia
G Haemolytic disease of the newborn
H Paroxysmal nocturnal haemoglobinuria
I Microangiopathic haemolytic anaemia

5 A 1-day old baby has developed severe jaundice on the neonatal ward. The mother is rhesus negative and has had one previous pregnancy. Due to having her first baby abroad, she was not administered prophylactic anti-D.

A

5) G

Haemolytic disease of the newborn (G) occurs when the mother’s blood is rhesus negative and the fetus’ blood is rhesus positive. A first pregnancy or a sensitizing event such as an abortion, miscarriage or antepartum haemorrhage leads to fetal red blood cells entering the maternal circulation resulting in the formation of anti-D IgG. In a second pregnancy, maternal anti-D IgG will cross the placenta and coat fetal red blood cells which are subsequently haemolyzed in the spleen and liver. Therefore, anti-D prophylaxis is given to at-risk mothers; anti-D will coat any fetal red blood cells in the maternal circulation causing them to be removed by the spleen prior to potentially harmful IgG production.

Hereditary spherocytosis (A) and hereditary eliptocytosis are both auto- somal dominant disorders that result in RBC membrane defects and extravascular haemolysis.

Beta-Thalassaemia (C) results in defects of the globin chains of haemoglobin. As a consequence, there is damage to RBC membranes causing haemolysis within the bone marrow.
Pyruvate kinase deficiency (E) is an autosomal recessive genetic disorder that causes reduced ATP production within RBCs and therefore reduces survival.

Paroxysmal nocturnal haemoglobinuria (H; PNH) is a rare stem cell disorder which results in intravascular haemolysis, haemoglobinuria (especially at night) and thrombophilia. Ham’s test is positive.

How well did you know this?
1
Not at all
2
3
4
5
Perfectly
28
Q
  1. Obstetric haematology
    A 28-year-old woman in her 29th week of pregnancy comes to accident and emergency with epigastric pain, nausea and vomiting. She also complains that her hands and feet have been swelling up. On examination her blood pressure is 165/96, HR 125bpm, and she is apyrexial. She is noted to have yellowing of her sclera and right upper quadrant tenderness. Blood tests reveal an Hb of 10.1, platelets 96, WCC 11.3, LDH 820 (N 70–250), AST 115 (N 5–35), and ALT 102 (N 5–35). Her coagulation screen is normal and a blood film is reported as showing the presence of schistocytes. The most likely diagnosis is
A Hepatitis
B Thrombotic thrombocytopenic purpura
C Pre-eclampsia
D Acute fatty liver of pregnancy
E HELLP syndrome
A

E
‘HELLP’ syndrome (E) is a potentially fatal occurrence in pregnancy, characterized by a triad of features:

1 H – haemolysis
2 EL – elevated liver enzymes
3 LP – low platelet count

In a similar way to DIC, generalized activation of the clotting cascade is triggered which can only be terminated with delivery. Platelet consumption and MAHA occurs, and liver ischaemia can lead to periportal necrosis and, in severe cases, formation of a subcapsular haematoma which can rupture.

It usually presents in the third trimester, but can happen even up to a week after delivery. Often patients with HELLP have had pregnancy-induced hypertension or pre-eclampsia prior to its development. Common symptoms are often vague, and can include nausea and vomiting, epigastric pain, peripheral swelling, paraesthesia, headaches and visual problems. On examination patients may be noted to have peripheral oedema, upper abdominal tenderness, jaundice and hepatomegaly. Complications can include liver and renal failure, pulmonary oedema, DIC and placental abruption. Clotting studies may be normal as in this case, unless DIC has occurred. The only effective treatment is delivery, but other supportive treatment includes control of the hypertension, seizure prophylaxis and corticosteroid use.

Hepatitis (A) can result in jaundice, right upper quadrant tenderness and abnormal LFTs, but is less likely to cause a marked thrombocytopenia and the presence of schistocytes (indicating haemolysis) in an apyrexial patient as in this case. A leukocytosis is also more likely.

TTP (B) is characterized by the classic sextet of symptoms as described previously, but deranged LFTs are not typical of this condition.

Haemolysis and abnormal LFTs are also rare in pre-eclampsia (C), and mild thrombocytopenia is present in only 10–15 per cent of cases.

Acute fatty liver of pregnancy (D) is a life-threatening rare complication of pregnancy that can also present non-specifically with deranged LFTs, but is often accompanied by abnormal coagulation, leukocytosis and hypoglycaemia.

How well did you know this?
1
Not at all
2
3
4
5
Perfectly
29
Q
  1. Vitamin K dependent clotting factors
    A 56-year-old woman with known cirrhosis presents with falls. On examination she is clinically jaundiced and rectal examination reveals malaena. Blood tests reveal an INR of 2.2. She is diagnosed with decompensated chronic liver disease. Which of the following is not a vitamin K dependent clotting factor?
A Thrombin
B Factor VII
C Factor VIII
D Protein C
E Factor X
A

C
The vitamin K dependent clotting factors include II, VII, IX and X. Vitamin K is also required for the production for protein C, protein S and protein Z, although these are strictly not clotting factors, rather anticoagulant factors. Vitamin K is a fat soluble vitamin found in green leafy vegetables such as spinach, cabbage and cauliflower. It
is absorbed in the small bowel and is important in the production of functional clotting factors in the liver. This patient’s acute chronic liver failure has meant she is no longer producing functional clotting factors, represented as a raised INR.
Vitamin K is recycled in the liver and its oxidation is coupled with the post-translational modification of glutamate residues to form gamma-carboxyglutamate. Vitamin K is firstly reduced by vitamin K epoxide reductase to form vitamin K hydroquinone. This reduced form is oxi- dized by vitamin K dependent carboxylase to form vitamin K epoxide. This reaction is coupled with gamma-glutamyl carboxylase; the enzyme responsible for post-translational modification of the vitamin K depend- ent factors. Vitamin K epoxide is then reconverted to vitamin K by vitamin K epoxide reductase; thus completing the cycle. If the patient were to be given vitamin K metabolism antagonists, e.g. warfarin, the clotting factors produced would still be immunologically identical (these are also known as Proteins Induced by Vitamin K Absence/Antagonism – PIVKA) but would lack efficacy as they are unable to interact with calcium or platelet factor 3.

Factor VIII is not a vitamin K dependent clotting factor, it is synthe- sized by endothelium and sinusoidal cells of the liver and is found in the plasma as well as non-covalently bound to von Willebrand factor (vWF). It is classically described to be a part of the intrinsic pathway of the clotting cascade which is tested by the use of the prothrombin time. Factor VIII is a procofactor activated by thrombin during the amplification phase of the coagulation cascade. Once active, it binds to factor IXa on the platelet which together activate factor X with Va. Factors Va, Xa, phospholipids and calcium together form the prothrombinase complex which leads to a thrombin burst where thrombin production is rapidly amplified.

Thrombin (A) is the penultimate product of the coagulation cascade, its functions include cleaving fibrinogen to form fibrin, activating platelets, activating procofactors V and VIII and activating zymogens VII, XI and XIII. It also acts to control fibrinolysis by binding to endothelial-bound thrombospondin thus activating protein C (D) and protein S. This complex inhibits cofactors V and VIII and acts as a negative feedback control on the coagulation cascade.

Factor VII (B) is a vital part of the extrinsic clotting cascade which is activated by tissue factor, a factor expressed by damaged endothelial cells. Within this initiation phase, factor VIIa activates downstream factors IX and X. Factors Xa and Va bind to the damaged area provid- ing the base site of coagulation activity where the amplification phase begins. Here, the Xa/Va complex activates prothrombin to thrombin which then acts to activate XI, VIII and V. The end result is the pro-thrombinase complex; a structure consisting of factor Va heavy and light chains, Xa, phospholipids and calcium which explodes with throm- bin generating activity in order to produce fibrin rapidly and stabilize the platelet clot. This final phase is thus called the propagation phase.

How well did you know this?
1
Not at all
2
3
4
5
Perfectly
30
Q
  1. Deep vein thrombosis

A 46-year-old man presents with pain and swelling in the right calf 2 weeks after being fitted with a plaster cast to his leg after a fall. The calf is tender, erythematous and swollen. He is also a heavy smoker and slightly overweight. His admitting physician suspects a deep vein thrombosis (DVT) and books an ultrasound of the calf. A DVT is confirmed and 5mg warfarin is started the next day. Two days later, the same patient develops pain and swelling in the other calf, an ultrasound confirms a further DVT in the contralateral leg. What factor is least likely to contribute to the development of the second DVT?

A Smoking
B Warfarin
C Previous DVT
D Being slightly overweight
E Plaster cast
A

D
All of the factors except being slightly overweight probably directly contributed to this patient developing a second deep vein thrombosis. The risk factors for developing venous thrombosis may be categorized to mechanisms affecting the blood vessel wall, the blood flow and the blood itself, i.e. Virchow’s triad.
Smoking (A) increases thrombotic risk by inducing endothelial damage and increasing thromboxane A2 production, which stimulates platelet aggregation, and perhaps increasing platelet dependent thrombin generation. The link between smoking and venous thrombosis is well established although the risk reduction over time once someone quits is not fully understood.

Previous DVT (C) is probably one of the strongest risk factors for DVT with a five-fold increase over baseline risk. This, along with a recent fit- ting of a plaster cast (E) and associated immobility, represents the highest risk for this patient in developing a second DVT. Other important risk factors include major surgery, particularly involving the abdomen or lower limb, cancer, prothrombotic states, some chemotherapeutic agents, myocardial infarction and congestive heart failure, pregnancy and combined oral contraceptive pill.

Warfarin (B), a commonly used anticoagulant, probably contributed to
a second DVT in this situation. Warfarin antagonizes vitamin K epox-
ide reductase; a liver enzyme responsible for recycling vitamin K to its reduced state. Warfarin thus antagonizes the production of vitamin K dependent factors including factors II, VII, IX, X, protein C and protein S. The latter two are anticoagulant factors which provide a negative feedback on the coagulation cascade by inhibiting procofactor V and VIII activation. Protein C and S have a shorter half life than the other coagulant factors thus when their production is inhibited by warfarin, a state of transient hypercoagulability is formed in the first few days after starting warfarin. Normally, clinicians will cover this problem by the use of concomitant heparin until the therapeutic range is obtained. In this patient, the admitting physician unfortunately did not give any heparin, and thus transiently increased the thrombotic risk of this already high risk patient.

Although obesity is associated with risk of development of DVT, this man is described as slightly overweight (D). Thus, in comparison to the other risk factors presented, it probably represents the lowest attributable risk to the second DVT.

How well did you know this?
1
Not at all
2
3
4
5
Perfectly
31
Q
  1. Peripheral blood film
    A 32-year-old woman presents with generalized fatigue. Full blood count shows a reduced haemoglobin level and reduced mean corpuscular volume. A peripheral blood film has revealed iron deficiency anaemia. What features are most likely to be seen on her peripheral blood film?

A Hypochromic and microcytic red blood cells with anisopoikilocytosis and acanthocytes
B Hypochromic and microcytic red blood cells with hypersegmented neutrophils
C Hypochromic and microcytic red blood cells with anisopoikilocytosis and no evidence of basophilic stippling
D Hypochromic and microcytic red blood cells with Howell–Jolly bodies and basophilic stippling
E Hypochromic and macrocytic red blood cells with target cells, acanthocytes and Howell–Jolly bodies

A

C
Features of iron deficiency anaemia are hypochromic (pale) and microcytic (small) red blood cells. Poikilocytes are red blood cells that are abnormally shaped. When there are variations in shape and size, it is known as anisopoikilocytosis. Basophilic stippling (aggregation of ribosomal material) is absent in iron deficiency and present in beta-thalassaemia trait and lead poisoning.

In megaloblastic anaemia, there is impaired DNA synthesis and this can be caused by B12 deficiency, folate deficiency and drugs. Here the features are the characteristic hypersegmented neutrophils and macrocytic red blood cells.

In hyposplenism, there is presence of target cells known as codocytes (red blood cells that have a high surface area:volume ratio).

Acanthocytes (spiculated blood cells/spur cells) and Howell–Jolly bodies (nuclear remnants visible in red cells) are also present in the hyposplenism picture.

How well did you know this?
1
Not at all
2
3
4
5
Perfectly
32
Q
  1. Thrombocytopenia
    A 54-year-old man presents with haematemesis. He has known varices and is currently vomiting large amounts of bright red blood. The admitting doctor takes some blood for fast analysis and confirms a haemoglobin of 4g/dL. The patient’s haematemesis continues and he is transfused a total of 20 units of blood and 8 units of fresh frozen plasma in the next 24 hours. The patient underwent gastroscopy which revealed bleeding oesophageal varices which were successfully treated by endoscopic banding.

His post-transfusion bloods are the following:
Hb 9.2g/dL
White cells 8.0×109/L
Platelets 57×109/L
Prothrombin time normal
Activated partial thromboplastin time normal
Fibrinogen >1.0g/L

What is the most likely cause of his thrombocytopenia?

A Disseminated intravascular coagulopathy
B Alcohol excess
C Massive blood transfusion
D Megaloblastic anaemia
E Hypersplenism
A

C
Although all of the given options are causes of thrombocytopenia, the most likely cause in this patient is massive blood transfusion without replacement of platelets (C). Massive blood loss may be defined as losing one’s entire circulating blood volume in 24 hours. Other definitions include losing 50% of one’s blood volume in 3 hours or a rate of loss of greater than or equal to 150mL/min. This patient has been transfused 20 units of blood in the space of 24 hours, thus fulfilling the criteria for massive haemorrhage. Massive transfusion has its own par- ticular complications, including thrombocytopenia. This is because this patient was only given packed red cells and fresh frozen plasma. These two blood products contain very few platelets and in general, a platelet count of around 50×10/9L is to be expected when approximately 2 blood volumes have been replaced, as is the case in this patient. In this situation, the expert consensus is to keep the platelet level above 50×109/L, but there is marked interindividual variation therefore some consider using 75×109/L as the trigger value for platelet transfusion.

Disseminated intravascular coagulopathy (DIC) (A) is a feared complication of massive blood transfusion and carries with it a high mortality. It can be thought of as the loss of haemostatic control resulting in consumption of coagulant factors, platelets and fibrinogen. Widespread clotting ensues with microvascular structures becoming ischaemic, resulting in potential end organ failure. Once coagulation factors and platelets are depleted bleeding becomes apparent making this disorder a concomitant bleeding and clotting problem. Those at particular risk include patients with prolonged hypoxia or hypovolaemia with cerebral or extensive muscle damage, or those who become hypothermic from infusion of cold resuscitation fluids. It is biochemically detected by a rising prothrombin time, activated thromboplastin time in excess of that expected by dilution together with significant thrombocytopenia and low fibrinogen (<1.0g/L). This is not the case with this patient, although he should be monitored closely to look out for this complication.

Alcohol excess (B) can cause thrombocytopenia; it is a direct bone marrow suppressant thereby inhibiting megakaryocyte development and platelet production.

Cirrhosis, of any aetiology including alcohol, can cause portal hypertension thus causing splenomegaly and a potential hypersplenism (E). The normal human spleen contains about one-third of the circulating platelets, if it engorges in size due to portal hypertension it may sequester more platelets; this is the difference between hypersplenism (increased function) and splenomegaly (increased physical size). This is less likely in this case given the massive haemorrhage and the lack of clinical evidence of splenomegaly in the question, despite there being evidence of portal hypertension as there are oesophageal varices present.
Megaloblastic anaemia (D) is caused by B12 or folate deficiency which is classically macrocytic in nature, although the mean cell volume will be difficult to interpret now given the patient has had a blood transfusion. It is most commonly caused by pernicious anaemia, an autoimmune condition where antibodies are directed against stomach parietal cells or intrinsic factor. The blood film classically shows meg- aloblasts – nucleated red blood cells along with polychromasia (where red cells have multiple colours due to premature release from bone marrow), basophilic stippling (peripheral dots which represent rRNA and is always pathological) and Howell–Jolly inclusion bodies (clus- ters of DNA within erythrocytes). This could potentially be true in this patient, but it is not the most likely answer given the circumstances of massive blood loss.
How well did you know this?
1
Not at all
2
3
4
5
Perfectly
33
Q
  1. Erythrocyte sedimentation rate
    Which of the following is not often associated with a very high (>100mm/hour) erythrocyte sedimentation rate (ESR)?
A Myeloma
B Anaemia
C Leukaemia
D Aortic aneurysm
E Malignant prostatic cancer
A

B

ESR is a commonly used laboratory test to detect the presence of inflammation in general. It is performed by adding a sample of anticoagulant to a blood sample and adding this mixture to a calibrated vertical tube (Westergren tube). As the red cells fall with gravity and accumulate, they lie in the bottom of the tube, and are called sediment. The rate at which they accumulate is therefore the ESR.

Factors which influence the ESR include age, sex and pathological processes which increase plasma proteins or the number of red cells. Women generally have a higher ESR than men and it also increases with age. Depending on the exact reference range for your particular lab, women and men over 50 can have an ESR of up to 30 and 20mm/ hour, respectively, and still be normal. Conditions which increase plasma proteins such as fibrinogen, acute phase proteins and immunoglobulins can increase the ESR as these proteins reduce the ionic resist- ance between erythrocytes leading to an increased fall rate. They also promote rouleaux formation of erythrocytes which is the characteristic stacking of erythrocytes seen under the microscope. The most important protein to promote rouleaux formation is fibrinogen. The number of red cells in a given volume also influences ESR; in severe anaemia ESR is falsely raised as the reduced ionic repulsion between erythrocytes allow faster sedimentation. However, this rarely leads to an ESR of >100mm/ hour, making anaemia (B) the correct answer. The other conditions listed can all raise ESR above 100mm/hour.

Myeloma (A) and leukaemia (C) do this by the production of increased plasma proteins including immunoglobulins which promote rouleaux formation as well as reduce ionic erythrocyte repulsion.

Aortic aneurysms (D) can cause a very raised ESR, particularly when they are of the inflammatory type. Patients with chronic abdominal pain, weight loss, raised ESR with a known abdominal aneurysm should prompt the thought of an inflammatory aneurysm subtype. In these patients the inflammatory process sometimes encases the nearby ureters causing obstruction and eventually hydronephrosis.

Malignant prostate cancer (E) raises ESR by virtue of raising fibrinogen levels in the blood. Quantative in vitro studies have found a direct relationship between fibrinogen concentration and ESR.

Fibrinogen is an important part of the clotting cascade; its activation to fibrin is important in binding to platelets and stabilizing the platelet plug to maintain haemostasis. As mentioned, fibrinogen increases rouleaux formation as well as reducing ionic repulsion between erythrocytes, thus increasing ESR.

Sometimes patients present with a persistently raised ESR but a normal C reactive protein – another marker of inflammation which rises and falls more acutely. There are a few important conditions to note with this configuration of test results: systemic lupus erythematosus, multiple myeloma, lymphoma, anaemia and pregnancy.

How well did you know this?
1
Not at all
2
3
4
5
Perfectly
34
Q
  1. Polycythaemia

A 62-year-old man presents with shortness of breath. This has been gradually getting worse for the last few years and is associated with chronic productive cough. He is a heavy smoker. His chest X-ray reveals a hyperexpanded chest with no other abnormalities. His bloods tests are normal except for a raised haemoglobin and raised haematocrit. What is the most likely cause for this?

A Polycythaemia rubra vera
B Idiopathic erythrocytosis
C Secondary polycythaemia
D Gaisbock’s disease
E Combined polycythaemia
A

E Combined polycythaemia (E), also known as smoker’s polycythaemia, has multiple aetiological factors. Cigarettes contain high concentrations of carbon monoxide gas which bind avidly to haemoglobin, thus displacing oxygen. This leads to increased erythropoietin (EPO) secretion from the hypoxic renal interstitium. EPO promotes erythrocyte proliferation and differentiation and prevents their apoptosis in the bone marrow, thus increasing red cell mass. Smoking is also a significant risk factor for chronic obstructive pulmonary disease, which is what this man suffers from. The obstructed airways reduce oxygen delivery to the alveoli and pulmonary vessels they supply thus causing a reduction of oxygen supply furthering the hypoxia. Finally, smokers also have an associated reduced plasma volume, thus increasing the relative concentration of haemoglobin. This is therefore ‘combined’ because of the presence of both increased red cell mass and reduced plasma volume.

Polycythaemia rubra vera (PRV) (A) is a chronic myeloproliferative dis- order characterized by a V617F point mutation in exon 14 of the JAK2 gene (E). It is present in 95–97% of patients with PRV, but the finding of this mutation is not specific to this condition (it also occurs in a substantial proportion of patients with essential thrombocythaemia and myelofibrosis). Crucially, these patients will have an increased red cell mass. It is important to realize that a raised haemoglobin, raised haematocrit or raised red blood cell count alone is not the same as a raised red cell mass. Haemoglobin may be raised with relative deficiency of plasma (i.e. relative or apparent polycythaemia, historically known as Gaisbock’s disease (D)). This is also the same with haematocrit, which is a measurement of the proportion of a centrifuged test tube red cells occupy compared with the entire sample. If there is a relative deficiency of plasma, e.g. secondary to dehydration, there is a relative increase in the haematocrit. The red cell mass is an absolute measure and is assessed by isotope dilution studies. This is sometimes used to differentiate between true and apparent polycythaemia.

Idiopathic erythrocytosis (B) is the label given to those with polycythaemia secondary to JAK2 mutation, but not with the V617F exon 14 mutation, e.g. exon 12 mutations.

Secondary polycythaemia (C) is where there are circulating plasma factors stimulating erythropoeisis, usually EPO but sometimes anabolic steroids (e.g. testosterone). It can also be secondary to an EPO secreting tumour – the 5 most common of which include hepatocellular carcinoma, renal cell carcinoma, haemangioblastoma, phaechromocytoma and uterine myomata. Oxygen sensitive EPO response may be appropriate, for example in chronic hypoxia when living at altitude or inappropriate, e.g. post transplant erythrocytosis where other hormones act to increases erythropoiesis and the EPO concentration is not elevated.

How well did you know this?
1
Not at all
2
3
4
5
Perfectly
35
Q
  1. Haemolysis
    A 25-year-old black man develops jaundice and dark red urine 2 days after starting primaquine, an anti-malarial. His blood tests reveal a macrocytic anaemia with raised bilirubin and urine dipstick is positive for blood. A peripheral blood film reveals ‘bite cells’ and Heinz bodies. The most likely diagnosis is:
A Hereditary spherocytosis
B Glucose-6-phosphate dehydrogenase deficiency
C Paroxysmal nocturnal haemoglobinuria
D Microangiopathic haemolytic anaemia
E Autoimmune haemolytic anaemia
A

B

G6PD deficiency (B) (also known as favism) is an X-linked condition where the lack of this enzyme increases the oxidative damage sustained by red blood cells. It is part of the pentose phosphate pathway which maintains levels of reduced glutathione – an important erythrocyte anti- oxidant. People with this condition have erythrocytes with less reduced glutathione and are thus more sensitive to oxidative stress resulting in haemolysis. There are many variants of G6PD of differing severity. It is unlike some X-linked conditions where women can also be affected if they are homozygous. Precipitating factors include anti-malarials, pri- maquine, nitrofurantoin, dapsone, sulphonylureas and sulphonamides. Its alternate name, favism, relates to the fact that fava beans (broad beans) can trigger a haemolytic attack. The haemolysis released intracellular haemoglobin causing jaundice in this patient and haemoglobinuria, which caused a positive dipstick result. It is important to note that the differential for a positive dipstick for blood includes haemoglobinuria and myoglobinuria. Macrocytosis occurs from the raised reticulocyte production from increased bone marrow activity. Heinz bodies are seen due to the denatured haemoglobin which gets removed by macrophages in the spleen leaving ‘bite’ cells. A G6PD assay is useful in this patient but less so in the acute setting as the new reticulocytes can contain normal G6PD levels, thus giving a false negative result.

Hereditary spherocytosis (A) is an autosomal dominant disorder char- acterized by erythrocyte cytoskeletal abnormalities. The most common is due to a spectrin protein defect which is responsible for binding to protein 4.1, thus joining the intracellular cytoskeleton to the external membrane based proteins. The abnormal cell structure results in reduced erythrocyte flexibility giving it its characteristic shape but also making it more vulnerable to splenic sequestration and extravascular haemolysis. The erythrocyte vulnerability is exploited by the osmotic fragility test which strains the erythrocyte in varying solution concentrations to determine how fragile the cells are.

Paroxysmal nocturnal haemoglobinuria (C) is an acquired clonal stem cell disorder characterized by an abnormal erythrocyte sensitivity to the lytic complement pathway. There is a deficiency of glucosyl phosphatidyl inositol (GPI) linked proteins namely CD55 (decay-accelerating factor) and CD59 (membrane inhibitor or reactive lysis). The nocturnal nature of haemolysis is thought to be due to the relative acidosis created during sleep, thus causing haemolysis and red urine in the morning. There is also an association with thrombosis particularly Budd–Chiari syndrome (hepatic vein thrombosis).

Microangiopathic haemolytic anaemia (D) refers to haemolysis of red cells caused by narrowing microvasculature seen in patients with mechanical heart valves, disseminated intravascular coagulopathy, thrombotic thrombocytopenic purpura or haemolytic uraemic syndrome.

Autoimmune haemolytic anaemia (E) occurs when autoantibodies directed at erythrocytes trigger haemolysis. The origin of the antibody might be due to a systemic disease (e.g. systemic lupus erythematosus, chronic lymphocytic leukaemia), infection (mycoplasma related cold agglutinins) or drugs (penicillins). Primaquine does not often cause an autoimmune haemolysis.

36
Q
  1. Secondary polycythaemia
    What are the likely laboratory findings for a patient with renal cell carcinoma with secondary polycythaemia who is not dehydrated?

A Normal red cell count, normal red cell mass, increased erythropoietin concentration
B Increased red cell count, increased red cell mass, increased erythropoietin concentration
C Decreased red cell count, decreased red cell mass, normal erythropoietin concentration
D Increased red cell count, decreased red cell mass, increased erythropoietin concentration
E Decreased red cell count, increased red cell mass, decreased erythropoietin concentration

A

B
This question tests your understanding of erythropoeisis physiology and your understanding of laboratory measurements in a standard full blood count analysis. Red cell count is measured as the number of erythrocytes in a quantum of plasma, whereas red cell mass is determined by isotope studies quoted as mL/kg. It is a measure of absolute red cell mass and is therefore not affected if someone is dehydrated, for example, where the relative plasma volume is reduced giving a falsely high red cell concentration. There are many situations where the red cell concentration and red cell mass do not parallel each other, e.g. vomiting, diarrhoea or overuse of diuretics. If a patient has increased red cell concentration this may therefore be absolute or relative – the latter being secondary to reduced plasma volume thus making the polycythaemia secondary to haemoconcentration. Absolute polycythaemia may be primary or secondary. In this case there is secondary polycythaemia where the renal cell carcinoma is inappropriately producing too much EPO, thus overstimulating bone marrow erythropoeisis. Secondary polycythaemia is not always inappropriate – people with cyanotic heart disease, lung disease, haemoglobinopathies with high oxygen affinity or those living at altitude can get appropriate secondary polycythaemia as a physiological response to chronic hypoxaemia.

If polycythaemia were to be primary, then the EPO levels would be low with the other parameters being high. This would be the case in patients with polycythaemia rubra vera where virtually all of these patients have a V617F gene mutation of exon 14 of the Janus Kinase 2 enzyme. These patients are at risk of thrombosis due to the marked increased in red cell mass, and development of hyperuricaemia from high cell turn- over. Patients also develop high basophil and eosinophil numbers – the increased histamine release from the former may results in intense pruritus and peptic ulceration.

37
Q
  1. von Willebrand’s disease (1)
    von Willebrand’s disease is characterized by abnormal platelet aggregation when they are exposed to:
A Streptomycin
B Aspirin
C Fibrinogen
D Collagen
E Ristocetin
A

E
von Willebrand’s disease (vWD) is characterized by a quantitive or qualititative defect in von Willebrand factor (vWF). Ristocetin, an antibiotic no longer used clinically, causes vWF to bind the platelet receptor glycoprotein Ib (GlpIb) through an unknown mechanism. If ristocetin is added to platelets with defective vWF or defective GlpIb (called Bernard–Soulier syndrome) then platelet aggregation does not occur. It will occur, however, with other pro-aggregative factors including colla- gen (D) and fibrinogen (C). If vWF or GlpIb is absent, aggregation does not occur with collagen as there is no molecular link between collagen and the platelet. However, this is the case with all patients with vWF.

Furthermore, cryoprecipitate which contains vWF will correct defects in vWD but not in Bernard–Soulier syndrome.

Platelet disorders may be inherited or acquired. Inherited disorders include Glanzmann’s thrombasthenia (an inherited lack of GlpIIb/IIIa) where fibrinogen cannot cross-link platelets during the initial platelet aggregative stage of thrombosis. Understanding these receptors and their importance in platelet aggregation has led to the development of powerful antiplatelet medications including adciximab, eptifibatide and tirofiban. Other inherited platelet diseases include storage pool diseases, e.g. grey platelet syndrome, Quebec platelet disorder, Hermansky–Pudlak syndrome and Chediak–Higashi syndrome. These refer to defects of the alpha and dense granules in the platelet which are released to promote platelet aggregation. Acquired platelet defects include aspirin (B) and uraemia. Aspirin irreversibly inhibits cyclooxygenase thus reducing levels of thromboxane A2, a powerful aggregative factor.

38
Q
  1. Myeloproliferative disease
    An 18-month-old child with Down syndrome presents with recurrent infections and petechial bleeding. A blood film was analyzed showing a particular distinctive feature of haematological malignancy. What is the most likely diagnostically helpful finding seen in this patient?
A Smudge cell
B Reed Sternberg cell
C Auer rod
D Pelger Huet anomaly
E Hairy cell
A

C
The Auer rod (C) is pathognomonic of acute myeloblastic leukaemia (AML). Children with Down syndrome are at higher risk of this disease due to chromosome 21 duplication where a ‘dosage’ effect is theorized to increase the expression of proto-oncogenes. This may also explain the increased risk of AML in Warkany syndrome type 2 (trisomy 8). Another dosage effect example, also in Down syndrome, is the increased risk of Alzheimer’s disease with beta amyloid, which accumulates in Alzheimer’s disease and is coded for on chromosome
21. Epidemiologically, children with Down syndrome are more likely to get AML than ALL in the first 3 years of their life, but thereafter are more likely to get ALL, similar to those without Down syndrome. Auer rod’s are pathognomonic for AML and are found in the cytoplasm. They represent stacked granules in myeloblasts and are azurophilic. They are particularly common in the M3 subtype of AML (according to the French American British classification).

Smudge cells (A) are seen in chronic lymphocytic leukaemia, the most common leukaemia and generally a disease of elderly people. Smudge cells represent the fragility of B-cell CLL upon mechanical manipulation and considered characteristic of CLL. CLL is characterized by the failure of B-cell apoptosis and the cells have undergone cell cycle arrest in the G0/G1 phase. White cell counts of over 100 are typical in patients who may be completely asymptomatic.

Reed–Sternberg cells (B) are pathognomonic for Hodgkin’s lymphoma. These cells are actually uncommonly found when looking at histological samples from patients with Hodgkin’s lymphoma. Recent advances looking at the origin of the Reed–Sternberg cell reveal it is most likely derived from germinal centre B cells, but they have characteristically lost their phenotypic features. Furthermore, routine stains for leukocyte common antigen, T cell and B cell markers are usually negative.

The Pelger–Huet anomaly (D) is caused by a benign dominantly inherited defect of terminal neutrophil differentiation. It is due to a mutation in the lamin-B receptor gene. The nucleus is bilobed and dumbbell- shaped with a narrow bridge connecting the two. This anomaly can also be acquired secondary to colchicine and sulphonamides which is revers- ible (so called pseudo-Pelger cells).

Hairy cells (E) are found in hairy cell leukaemia. This is an uncommon chronic B-cell lymphoproliferative disorder and is so named due to the prominent irregular cytoplasmic projections. It is now considered an indolent non-Hodgin lymphoma. It can present with splenomegaly, constitutional symptoms, bone marrow failure or incidentally with abnormal blood counts. The nuclei are variable in configuration and the cytoplasmic outline is often indistinct but best seen with phase-contrast microscopy.

39
Q
  1. Blood transfusion (3)
    An 83-year-old woman with myelodysplasia is found to have a haemoglobin of 6.2 on admission. She is transfused two units of blood, and is discharged 2 days later. Six days after her admission her carer calls the GP with concerns that she is feverish and her skin looks slightly yellow. She is readmitted to hospital where blood tests reveal the following: bilirubin 35, ALT 15 (N 5–35), ALP 82
    (N 20–140), Hb 7.3g/dL, platelets 264×109/L. The most likely diagnosis is:
    A Febrile haemolytic transfusion reaction
    B Hepatitis B
    C Graft versus host disease
    D Post-transfusion purpura
    E Delayed haemolytic transfusion reaction
A

E
Delayed haemolytic transfusion reactions (E) can occur more than 24 hours after a transfusion is given. They occur when patients are sen- sitized from previous transfusions or pregnancies, and therefore have antibodies against red cell antigens which are not picked up by routine blood bank screening if they are below the detectable limits. The most frequent causes are the antibodies of the Kidd (Jk) and Rh systems.
Clinical features might include falling haemoglobin concentration, a smaller rise in haemoglobin than expected following a transfusion as in this case, fever, jaundice and rarely haemoglobinuria or renal failure. A blood film may show a raised reticulocyte count. Management of these reactions includes monitoring renal function, sending a repeat group and antibody screen and cross-match and further transfusion if needed. The blood bank should be notified too, and further specific treatment might not be needed unless renal failure develops.
Febrile haemolytic transfusion reactions (A) typically occur less than
24 hours after the transfusion. These reactions are thought to be due to antibodies in the patient reacting with white cell antigens in the donor blood, or due to cytokines which build up in the blood products during storage. These reactions usually only warrant slowing the transfusion, and giving an anti-pyretic if needed. Hepatitis B (B) can occur after a blood transfusion, but blood products are usually screened for this virus as well as for the hepatitis C antibody and RNA, HIV antibody, HTLV antibody, and syphilis antibody. A high ALT would be expected if the patient had been infected with a hepatitis virus. Graft versus host dis- ease (C) is a rare form of delayed reaction which can occur in immuno- suppressed patients, where lymphocytes from donor blood can attack the host. This can result in liver failure, diarrhoea, skin rashes and bone marrow failure. Post-transfusion purpura (D) is a rare but potentially lethal reaction which occurs 5–9 days after a transfusion. Patients can develop a severe thrombocytopenia with bleeding. Treatment is usually with IV immunoglobulin therapy.

40
Q
3. The peripheral blood film
A Anisocytosis
B Howell–Jolly bodies
C Heinz bodies
D Rouleaux formation
E Spherocytes
F Target cells
G Cabot rings
H Pappenheimer bodies 
I Tear-drop cells

1 A 34-year-old man, who has a past medical history of splenectomy following splenic trauma, presents to his GP with malaise 2 weeks after returning from abroad. Routine blood results are found to be normal but a blood film demonstrates inclusions within erythrocytes.

A

1) B

Howell–Jolly bodies (B) are nuclear DNA remnants found in circulating erythrocytes. On haematoxylin and eosin stained blood film they appear as purple spheres within erythrocytes. In healthy individuals erythrocytes expel nuclear DNA during the maturation process within the bone marrow; the few erythrocytes containing Howell–Jolly bodies are removed by the spleen. Common causes of Howell–Jolly bodies include splenectomy secondary to trauma and autosplenectomy resulting from sickle cell disease.

Rouleaux (D) formation describes the stacks of erythrocytes that form in high plasma protein states, for example, multiple myeloma.

Spherocytes (E) are caused by hereditary spherocytosis (defect in mem- branous proteins, for example spectrin), which leads to haemolytic anaemia.

Target cells (F) are erythrocytes with a central area of staining, a ring
of pallor and an outer ring of staining. They are formed in thalassaemia, asplenia and liver disease.

Pappenheimer bodies (H) are granules of iron found within erythrocytes. Causes include lead poisoning, sideroblastic anaemia and haemolytic anaemia.

41
Q
3. The peripheral blood film
A Anisocytosis
B Howell–Jolly bodies
C Heinz bodies
D Rouleaux formation
E Spherocytes
F Target cells
G Cabot rings
H Pappenheimer bodies 
I Tear-drop cells

2 A 66-year-old man has a gastroscopy and colonoscopy following a blood test which demonstrated a microcytic anaemia. The patient had complained of tiredness and significant weight loss over a 1-month period.

A

2) A

Anisocytosis (A) is defined as the variation in the size of circulating erythrocytes. The most common cause is iron deficiency anaemia (IDA), but thalassaemia, megaloblastic anaemia and sideroblastic anaemia are all causative. As well as blood film analysis, anisocytosis may be detected as a raised red cell distribution width (RDW), a measure of variation in size of red blood cells. In the case of IDA, anisocytosis results due to deficient iron supply to produce haemoglobin.

Rouleaux (D) formation describes the stacks of erythrocytes that form in high plasma protein states, for example, multiple myeloma.

Spherocytes (E) are caused by hereditary spherocytosis (defect in mem- branous proteins, for example spectrin), which leads to haemolytic anaemia.

Target cells (F) are erythrocytes with a central area of staining, a ring
of pallor and an outer ring of staining. They are formed in thalassaemia, asplenia and liver disease.

Pappenheimer bodies (H) are granules of iron found within erythrocytes. Causes include lead poisoning, sideroblastic anaemia and haemolytic anaemia.

42
Q
3. The peripheral blood film
A Anisocytosis
B Howell–Jolly bodies
C Heinz bodies
D Rouleaux formation
E Spherocytes
F Target cells
G Cabot rings
H Pappenheimer bodies 
I Tear-drop cells

3 A 36-year-old woman presents to her GP after a 1-month history of tiredness and recurrent chest infections. Blood tests reveal a pancytopenia and a subsequent bone marrow aspirate reveals a dry tap.

A

3) I

Tear-drop cells (I), also known as dacrocytes, are caused by myelofibrosis. The pathogenesis of myelofibrosis is defined by the bone marrow undergoing fibrosis, usually following a myeloproliferative disorder such as polycythaemia rubra vera or essential thrombocytosis. Bone marrow production of blood cells decreases resulting in a pancytopenia. The body compensates with extra-medullary haemopoiesis causing hepatosplenomegaly. Blood film will demonstrate leuko-erythroblasts, tear-drop cells and circulating megakaryocytes. Bone marrow aspirate is described as a ‘dry and bloody’ tap.

Rouleaux (D) formation describes the stacks of erythrocytes that form in high plasma protein states, for example, multiple myeloma.

Spherocytes (E) are caused by hereditary spherocytosis (defect in mem- branous proteins, for example spectrin), which leads to haemolytic anaemia.

Target cells (F) are erythrocytes with a central area of staining, a ring
of pallor and an outer ring of staining. They are formed in thalassaemia, asplenia and liver disease.

Pappenheimer bodies (H) are granules of iron found within erythrocytes. Causes include lead poisoning, sideroblastic anaemia and haemolytic anaemia.

43
Q
3. The peripheral blood film
A Anisocytosis
B Howell–Jolly bodies
C Heinz bodies
D Rouleaux formation
E Spherocytes
F Target cells
G Cabot rings
H Pappenheimer bodies 
I Tear-drop cells

4 A 3-week-old neonate is found to have prolonged jaundice with serious risk of kernicterus. Blood film demonstrates the presence of ‘bite cells’ as well as inclusions within erythrocytes.

A

4) C

Heinz bodies (C) are inclusion bodies found within erythrocytes that represent denatured haemoglobin as a result of reactive oxygen species. Heinz bodies are most commonly caused by erythrocyte enzyme deficiencies such as glucose-6-phosphate dehydrogenase (G6PD) deficiency, which may present in neonates with prolonged jaundice and NADPH deficiency (leading to accumulation of hydrogen peroxide), as well as chronic liver disease and alpha-thalassaemia. Damaged erythrocytes are removed in the spleen by macrophages leading to the formation of ‘bite cells’.

Rouleaux (D) formation describes the stacks of erythrocytes that form in high plasma protein states, for example, multiple myeloma.

Spherocytes (E) are caused by hereditary spherocytosis (defect in mem- branous proteins, for example spectrin), which leads to haemolytic anaemia.

Target cells (F) are erythrocytes with a central area of staining, a ring
of pallor and an outer ring of staining. They are formed in thalassaemia, asplenia and liver disease.

Pappenheimer bodies (H) are granules of iron found within erythrocytes. Causes include lead poisoning, sideroblastic anaemia and haemolytic anaemia.

44
Q
3. The peripheral blood film
A Anisocytosis
B Howell–Jolly bodies
C Heinz bodies
D Rouleaux formation
E Spherocytes
F Target cells
G Cabot rings
H Pappenheimer bodies 
I Tear-drop cells

5 A 45-year-old woman with known Graves’ diseases presents to her GP with increased tiredness. She is found to have a megaloblastic anaemia.

A

5) G

Cabot rings (G) are looped structures found within erythrocytes which may be caused by megaloblastic anaemia, i.e. inhibition of erythrocyte production occurring as a result of reduced DNA synthesis secondary to vitamin B12 deficiency. Vitamin B12 deficiency is most commonly caused by intrinsic factor (a protein required for vitamin B12 absorption) deficiency as a result of pernicious anaemia. Pernicious anaemia is caused by antibody destruction of gastric parietal cells which produce intrinsic factor and may be associated with other autoimmune diseases.

Rouleaux (D) formation describes the stacks of erythrocytes that form in high plasma protein states, for example, multiple myeloma.

Spherocytes (E) are caused by hereditary spherocytosis (defect in mem- branous proteins, for example spectrin), which leads to haemolytic anaemia.

Target cells (F) are erythrocytes with a central area of staining, a ring
of pallor and an outer ring of staining. They are formed in thalassaemia, asplenia and liver disease.

Pappenheimer bodies (H) are granules of iron found within erythrocytes. Causes include lead poisoning, sideroblastic anaemia and haemolytic anaemia.

45
Q
  1. Lymphoma
    An 8-year-old African boy presents with a large jaw mass which has been growing rapidly over the last few weeks. A histological sample was taken and a classical ‘starry sky’ appearance was observed. The most likely diagnosis is:
A Follicular lymphoma
B Marginal zone lymphoma
C Burkitt’s lymphoma
D Diffuse large B cell lymphoma
E Mantle cell lymphoma
A

C

Burkitt’s lymphoma (C) is one of the most aggressive malignancies known to man. It is a type of non-Hodgkin’s lymphoma (NHL) which arises from lymph node germinal centres. It is associated with Epstein–Barr virus and there are three subtypes – endemic, sporadic and immunodeficiency related. It is associated with translocation and dysregulation of the c-myc gene on chromosome 8 including t(8;14), t(2;8) and t(8;22). Histologically, there is profound proliferation. The starry sky appearance reflects islands of macrophages ingesting necrotic tumour cells as they have outgrown their own blood supply. Clinically, the endemic form affects younger men and classically presents with a jaw mass which spreads to extranodal sites including mesentery, ovary, testis, bone marrow and meninges.

Follicular lymphoma (A) is the most common indolent NHL. It is defined as a lymphoma of the follicle centre B cells (centrocytes). It usually pre- sents with painless adenopathy with a prolonged history of waxing and waning lymphoadenopathy. Histologically, a nodular growth pattern with a mixture of centrocytes and centroblasts is characteristic. The proportion of centroblasts found determines the tumour grade which has therapeutic implications. These cells are CD10 and BCL-6 positive in the majority. It is usually indolent but can transform into a high grade lymphoma.

Marginal zone lymphoma (B) arises mainly at extranodal sites due to chronic antigenic stimulation and malignant transformation. It is a B-cell NHL arising from marginal cells in the lymph node. There are clinically three subtypes: extranodal marginal zone B-cell lymphoma of mucosa associated lymphoid tissue (MALT); nodal marginal zone B-cell lymphoma; and splenic marginal zone B-cell lymphoma. MALT lymphoma can occur in the stomach, lung, thyroid, breast, synovium and lacrimal or salivary glands. Associated conditions with MALT lympho- ma include autoimmune conditions (e.g. Sjögren’s disease, Hashimoto’s thyroiditis, relapsing polychondritis) and Helicobacter pylori infection in the stomach. In fact, early treatment of H. pylori infection can induce regression.

Diffuse large B-cell lymphoma (D) is a subtype of B-cell NHL which usually affects elderly men presenting with lymphadenopathy and con- stitutional ‘B’ symptoms (weight loss, fever, drenching night sweats) in one-third. It is the most common histological subtype of NHL. The diagnosis is usually made with an excisional biopsy demonstrating pan-B cell markers, e.g. CD20, and CD79a. Histologically, there is effacement of normal architecture by sheets of atypical lymphoid cells. The cell of origin is the centroblast. There is some overlap with Burkitt’s lymphoma as diffuse large B-cell lymphoma can have high proliferative indexes and t(8;14) mutations also seen in Burkitt’s.

Mantle cell lymphoma (E) is another B-cell NHL, whose precursor cell is the mantle cell in the pre-germinal area. It typically affects middle- aged men with lymphadenopathy and gastrointestinal tract involvement including spleen and Waldeyer’s ring. It usually presents with advanced disease and involves any region of the GI tract occasionally presenting as multiple intestinal polyps. Nuclear staining for cyclin D1 is present in 95% of cases and is strongly associated with the t(11;14) (q13;q32) mutation – a translocation between cyclin D1 locus and immunoglobulin heavy chain locus. This is not specific, however, for Mantle cell lymphoma and has been seen in multiple myeloma.

46
Q
  1. Hodgkin’s lymphoma

A 35-year-old Afro-American man presents with painless lymphadenopathy which he noticed after shaving. He denies any recent infections, fevers, weight loss or night sweats. A biopsy is performed which shows lymphocytic and histiocytic cells. A haematologist calls to confirm the diagnosis of non-classical Hodgkin’s lymphoma. Which subtype is this?

A Nodular sclerosis
B Mixed cellularity
C Nodular lymphocytic
D Lymphocytic rich
E Lymphocytic depleted
A
C
Nodular lymphocytic (C) Hodgkin’s lymphoma is often called the non- classical subtype of Hodgkin’s lymphoma. It is so called due to the atypi- cal nature of the Reed–Sternberg cell which characterizes all Hodgkin’s lymphomas. This cell is known as the lymphocytic and histiocytic cell or L&amp;H variant. Sometimes these cells are referred to as ‘popcorn’ cells because their nucleus resembles an exploded popcorn kernel. This subtype of HL accounts for 5 per cent of cases and has a bimodal age distribution – children and adults between the ages of 30 and 40. Unlike common HL, this type is more common in African American men compared with Caucasians in the US. Clinically, patients often present with peripheral lymphadenopathy without B symptoms (namely fever, night sweats and weight loss). It is generally thought of as a more indolent form of HL.

Nodular sclerosis classical HL (A) is characterized by at least a partially nodular grown pattern, with fibrous bands separating nodules. Mixed cellularity (B) classical HL is a heterogeneous group with a dif- fuse or vaguely nodular growth patter without band-forming sclerosis.

Lymphocytic rich (D) most commonly has a nodular growth pattern. In this subtype, cells resembling L&H variants may be present making the distinction from non-classical lymphoma more difficult. The background infiltrate consists predominantly of lymphocytes, with rare eosinophils or neutrophils. Lymphocyte depleted (E) subtype has a diffuse growth pattern and appears hypocellular with a lack of inflammatory cells. This subtype has a poor prognosis.

Questions about lymphoma are often difficult as detailed knowledge of subtypes is difficult to digest. The clinical clues in questions pointing to lymphoma include B symptoms, alcohol induced painful lymphadeno- pathy (although clinically rare), and the even rarer cyclical fever lasting 1–2 weeks termed the Pel–Ebstein fever.

47
Q
  1. Glucose-6-phosphate dehydrogenase deficiency
    A 17-year-old boy with glucose-6-phosphate dehydrogenase (G6PD) deficiency presents with tiredness and is noticed to be jaundiced. These features have developed since he was diagnosed with a chest infection 1 week ago. What is the most likely haematological finding?
A Positive direct antiglobulin test
B Low mean cell volume
C Reduced reticulocyte count
D Haemoglobinuria
E Increased haptoglobin concentration
A

D
This patient has glucose-6-phosphate dehydrogenase (G6PD) deficiency; a common X-linked condition where the reduction of G6PD function leads to haemolysis when erythrocytes are exposed to oxidative stress. Less commonly there is a chronic haemolysis when enzymatic activity is less than 10% of normal. Unlike some other X-linked conditions, women can be affected due to the random nature of X chromosome inactivation (lyonization) which leads to some cells being vulnerable to oxidative stress. G6PD is important in the pentose phosphate shunt which critically regenerates NADPH, a cofactor important in glutathione metabolism. Reduced glutathione is the primary buffer against oxidative stress. Haemoglobinuria (D) occurs due to intravascular haemolysis in the face of oxidative stress in a susceptible patient. Red cells undergo intravascular and extravascular haemolysis leading to haemoglobinaemia and haemoglobinuria. Common precipitants include intercurrent infection but also drugs, the most notorious of which are dapsone, primaquine and nitrofurantoin. Classically, haemolysis can be triggered by fava beans (hence its alternate name ‘Favism’) as well as naphthalene (found in moth balls and henna).

A positive direct antiglobin test (A) indicates an immunological cause for haemolysis. This is not true of G6PD deficiency as there is oxidative damage causing denatured erythrocyte proteins and haemolysis mostly via the reticuloendothelial system. This test, also known as the direct Coombs test, involves washing red cells and adding antiserum or monoclonal antibodies against various immunoglobulins (particularly IgG and C3d). A positive test results in red cell agglutination. It will therefore detect if antibodies or complement have bound to the red blood cell antigen. The indirect Coombs test is where antibodies or complement taken from the patient’s plasma will react to donor red blood cells. The indirect test is used for compatibility testing in blood transfusion.
A low mean cell volume often reflects decreased haemoglobin content and is often associated with a reduced mean cell haemoglobin. This is represented in the blood film as a hypochromic microcytosis. The most common cause is reduced iron availability (iron deficiency, anaemia of chronic disease or rarely copper deficiency). Other causes include reduced haem synthesis (lead poisoning, sideroblastic anaemia) or reduced globin production (thalassaemia or other haemoglobinopathies). Practically, an older patient with microcytic anaemia should be investigated for bowel cancer as this is a serious and common cause of chronic occult blood loss. With haemolysis, the mean cell volume normally increases – this is mainly due to the presence of reticulocytes which are larger than red cells. Thus one would expect a raised reticulocyte count and not a reduced count (C).

Finally, haptoglobin concentration is reduced during haemolysis, not increased (E). Haptoglobin circulates in the plasma and avidly binds to free haemoglobin to be later removed by the reticuloendothelial system. It is therefore reduced in intravascular haemolysis only. Extravascular haemolysis occurs mainly in the spleen where free haemoglobin is not released into the circulation and therefore haptoglobin levels are generally unchanged.

48
Q
  1. Anaemia (6)
    A 35-year-old Asian woman presents with tiredness. The full blood count shows:
Haemoglobin: 10.1g/dL (11.5–16.5) 
Platelet count: 160×109 (150–400×109) 
White cell count: 6.6×109 (4–11×109) 
Mean cell volume: 62fL (80–96fL)
Hb A2: 6.3 per cent (2–3 per cent)

Which of the following is the most likely diagnosis?

A Sickle cell disease
B Acute myeloid leukaemia
C Beta-Thalassaemia major
D Beta-Thalassaemia trait
E Hereditary spherocytosis
A

D
This woman presents with microcytic anaemia, the most common cause of which is iron deficiency. However, the mean cell volume is disproportionally reduced compared with the degree of anaemia indicating there might be a haemoglobinopathy present. The presence of increased Hb A2 confirms the diagnosis of beta-thalassaemia trait. Unfortunately the nomenclature surrounding beta-thalassaemia is relatively confusing but an attempt to clarify it will be made here. Firstly, thalassaemia is the reduction or absence of a type of globin gene. Normal haemoglobin is a tetramer of two alpha and two beta globin proteins. The ratio of alpha to non-alpha globin production is tightly controlled. There are two alpha genes located on chromosome 16, whereas only one beta gene on chromosome 11. beta-Thalassaemia implies a reduction or absence of the beta chain. Beta (0) thalassaemia refers to an absence of beta globin production. This encom- passes over 40 genetic mutations. Patients with this are often described as having beta-thalassaemia major (C), however, confusingly some patients can produce beta globin genes but to such a poor extent they behave very similar clinically, as if they had no production. Within the first year of life they have profound life-long transfusion dependent anaemia. This is therefore not compatible in this patient’s case.

Beta-Thalassaemia trait (D), also known as -thalassaemia minor, includes patients who have beta (+) thalassaemia. This means patients produce beta globin but to a lesser degree, and can produce some normal Hb A.
It is usually asymptomatic and picked up during blood tests for other reasons, such as in this case. Hb A2 is increased due to the imbalance of alpha globin production compared with beta globin production. A rare situation where HbA2 is not increased is in the setting of beta-thalassaemia trait with concomitant delta thalassaemia trait. Beta-Thalassaemia intermedia are patients who are compound heterozygotes of two thalassaemia variants, e.g. Beta-thalassaemia trait plus Hb E variant.

SCD (A) is caused by a valine to glutamic acid substitution in the 6th position of the beta globin gene. This reduces haemoglobin solubility once deoxygenated giving the characteristic sickle cell appearance and therefore causing microvascular occlusion resulting in ischemia and sickle crises. There is usually a chronic haemolysis with a normochromic normocytic anaemia. It may become microcytic if there is concomitant iron deficiency or coexistent - or -thalassaemia.

Hereditary spherocytosis (E) is a disorder of erythrocyte cytoskeleton giving a characteristic spherocyte appearance in the blood film. The mean cell volume is normal or low, but is confounded by the presence of reticulocytes which increases the average mean cell volume. This is reflected in an increased red cell distribution width (RDW) which is a standardized measure of the degree of anisocytosis. The most useful red cell index is the mean cell haemoglobin concentration (MCHC) which is normally elevated reflecting membrane loss and cell dehydration. A raised RDW with increased MCHC is very suggestive of this condition.

AML (B) is unlikely to present with these features and would normally have raised white cell counts with blasts as well as features of bone marrow failure.

49
Q
  1. Splenomegaly
    A 62-year-old man presents with bruising and tiredness. Examination reveals moderate splenomegly and his a reveal a normocytic anaemia with blood tests platelet count of 900×109/L, neutrophilia, basophilia, numerous myelocytes and 4% myeloblasts. The neutrophils have low leukocyte alkaline phosphatase levels. Which of the following is likely to be present in this patient?
A t(9;22)
B t (8;14)
C BCR-Abl fusion gene only
D V617F point mutation in JAK2
E 5q-Syndrome
A

A
This patient exhibits features of chronic myeloid leukaemia as evidenced by raised myeloid lineage cells including neutrophils, myelocytes and basophils. The neutrophils are morphologically normal but cytochemically different – a laboratory test sometimes used to dif- ferentiate between reactive or leukaemoid neutrophilia and CML is the leukocyte alkaline phosphatase. It is normal or high in the former, but characteristically low in CML. Absolute basophilia is a universal finding in CML, with absolute eosinophilia found in 90% of cases. A raised platelet count is also common in CML; a low platelet count, however, should make one reconsider the diagnosis, e.g. myelodysplastic syndromes.

Up to 95% of patients with chronic myeloid leukaemia have the Philadelphia chromosome – a fusion chromosome between the long arms of chromosomes 9 and 22 (A). The formation of the BCR-Abl fusion gene acts as a constitutively active tyrosine kinase, but the induction of leukaemogenesis is complicated and mediated through both tyrosine dependent and independent pathways. It is known, however, that the tyrosine kinase activity of the BCR-Abl gene is absolutely required for transformation. BCR-Abl fusion genes alone (C) can occur without the t(9;22) translocation but this is much less common. This Robertsonian translocation is worth remembering as it is frequently asked about in examinations.

The t(8;14) translocation occurs in Burkitt’s lymphoma: a highly aggressive lymphoma where cmyc, an oncogene, is under the influence of an immunoglobulin promoter, which is highly expressed. The V617F point mutation in JAK2 (D) is found in up to 99% of cases of polycythaemia rubra vera. JAK2 is involved in downstream processing of the erythropoietin receptor signalling. This mutation causes constitutive activation of downstream STATS molecules leading to uncontrolled gene transcription. 5q- is a subtype of myelodysplasic syndrome (MDS) in the WHO classification. The nomenclature means there is a deletion in the long arm of chromosome 5. It is the most common chromosomal abnormality seen in MDS (up to 15%).

50
Q
  1. Hodgkin’s lymphoma staging
    Which of the following patients has the worse prognosis?

A 25-year-old man with inguinal lymphadenopathy
B 25-year-old woman with mediastinal and inguinal lymphadenopathy
C 25-year-old woman with mediastinal and inguinal lymphadenopathy and night sweats
D 25-year-old woman with mediastinal and inguinal lymphadenopathy with 5% weight loss in last 6 months
E 25-year-old man with cervical and mediastinal lymphadenopathy

A

C
Ann Arbor staging system – stages are between I and IV either in the absence or presence of ‘B symptoms’.

A simplified version of the classification is as follows:

  • Stage I: involvement of a single lymph node region
  • Stage II: involvement of two or more lymph node regions on the same side of the diaphragm
  • Stage III: involvement of lymph nodes on both sides of the diaphragm
  • Stage IV: extranodal spread (not spleen however, this is taken as a lymph node)

The definition of B symptoms includes significant unexplained fever, night sweats or unexplained weight loss of over 10% during 6 months prior to diagnosis. The presence of B symptoms is denoted by a B subscript after the stage number, the absence is denoted by an A subscript. Patient A would therefore be classified as Ia, patient B as IIa, patient C as IIIb, patient D as IIIa, technically as she does not quite fulfil the 10% loss in 6 months and finally patient E as stage IIa.

NB. In Hodgkin’s lymphoma, the disease always spreads contiguously whereas in NHL this is not always the case. Further investigations for clinical staging include an upright chest X-ray, integrated positron emission tomography/CT of chest/abdomen/pelvis and sometimes a unilateral bone marrow aspirate and biopsy for those with stage III or IV with B symptoms.

51
Q
  1. Acute leukaemia
    A patient presents with acute promyelocytic leukaemia. What is the most likely mechanism of underlying leukaemogenesis?
A Telomere shortening
B Aberrant fusion of two genes
C Impaired protein degredation
D Over-expression of cellular oncogene
E Post-translational modification
A

B
Acute promyelocytic leukaemia is interesting for a number of reasons. There is a reciprocal translocation between the long arms of chromo- somes 15 and 17 giving the PML-RARA fusion gene (B). This links the retinoic acid receptor alpha (RARA) gene on chromosome 17 with the promyelocytic leukaemia (PML) gene on chromosome 15. RARA is a member of a family of retinoin-binding transcription factors that regulate gene expression. It heterodimerizes with retinoid X receptor (RXR) and binds to retinoic acid response elements to influence gene transcription. In the absence of retinoic acid, the RARA/RXR dimer interacts with another protein (nuclear corepressor) to repress gene transcription. Therefore, addition of retinoic acid stimulates gene transcription. In the setting of promyelocytic leukaemia, retinoic acid induces myeloid differentiation which is abnormally halted thus providing remission by encouraging cell differentiation rather than cell death. The second reason this type of leukaemia is interesting is its association with disseminated intravascular coagulopathy. The pathogenesis is not completely understood but recognizing it early is important as treatment with retinoic acid plus supportive therapy can lead to rapid improvement in the coagulopathy.

Telomeres (A) are nucleoprotein end caps on chromosomes which pro- vide protection of the chromosomal ends which are not copied fully during mitosis. It has been theorized that aging cells are more vulnerable to DNA damage due to shortened telomeres as they have undergone more mitosis and therefore have shorter telomeres.

The mechanisms for chromosomal mutations in leukaemia include:

  • chromosomal translocation where there is creation of a fusion gene or damage to a differentiation gene
  • chromosomal duplication where there is a potential dosage effect, e.g. Down syndrome and its association with AML
  • chromosomal loss or deletion can lead to the loss of tumour suppressor genes

Normally 2 types of gene mutation occur for acute leukaemia to occur – the first is type I mutation which promotes proliferation and survival whereas type II mutations block differentiation (which would normally be followed by apoptosis).

52
Q
  1. Treatment of chronic myeloid leukaemia
    A 64-year-old man presents with lethargy, weight loss and abdominal fullness. He is found to have chronic myeloid leukaemia. He is started on imatinib as part of the initial treatment to control his disease. What is the mechanism of action of imatinib?
A Proteosome inhibitor
B Tyrosine kinase inhibitor
C IL-6 inhibitor
D p53 inhibitor
E Human epidermal growth factor receptor 2 protein inhibitor
A

B
Imatinib is a tyrosine kinase inhibitor (B) and is used in the treatment of chronic myeloid leukaemia. It is a rational therapy which acts to inhibit the BCR-Abl tyrosine kinase thus blocking proliferation and inducing apoptosis in BCR-Abl positive cell lines. The BCR-Abl fusion is most commonly secondary to a balanced Robertsonian translocation between chromosomes 9 and 22. Imatinib is also used for gastrointestinal stromal tumour (GIST). Other tyrosine kinase inbitors include dasatinib and nilotinib. They do not cure CML but provide long-term control in the majority of patients, thus they are the initial treatment of choice for almost all newly diagnosed patients with CML.

Proteosome inhibitors (A) such as bortezomib inhibit 26S proteosome. This molecule is essential in the ubiquitin–proteosome pathway important for the degradation of intracellular proteins. Proteosome 26S inhibition leads to apoptosis via disruption of the NF kappa B pathway. Bortezomib is used in multiple myeloma and certain types of lymphoma.

IL-6 inhibitors (C) such as tocilizumab are biologic agents used in the treatment of rheumatoid arthritis. Blocking IL-6 prevents various immu- nological responses such as production of certain cytokines and acute phase proteins. It is not used in any malignancies.

Inhibitors of p53 (D) do not exist as p53 is a key inducer of apoptosis in cells with damaged DNA. Diseases where there is p53 mutation (e.g. Li–Fraumeni syndrome) result in a wide range of malignancies which develop at an unusually early age. It is also known as Sarcoma, Breast, Leukaemia and Adrenal gland (SBLA) cancer syndrome. Finally a human epidermal growth factor receptor 2 (HER2) protein antagonist, i.e. trastuzumab, is used in HER2 positive patients with breast cancer and sometimes in metastatic gastric cancer.

53
Q
  1. von Willebrand’s disease (2)
    A 16-year-old girl with mild von Willebrand’s disease is scheduled for a dental extraction. She has had one previously where she required two units of blood transfused. What is the most appropriate treatment for this patient prior to surgery?
A Cryoprecipitate
B Desmopressin
C Fresh frozen plasma
D Vitamin K
E Recombinant factor VIII concentrate
A

B

This woman has mild von Willebrand’s disease (vWD) which can be treated with desmopressin (B). There are three types of vWD – type I is a quantitative deficiency of von Willebrand Factor (vWF), type II is a qualitative defect in vWF whereas type III results in profound deficiency in vWF. There are four subtypes of type II vWF (2A, 2B, 2M and 2N). vWF is important in two ways; first it acts as a bridge between platelets and between platelets and subendothelial structures at the site of injury; and second it carries factor VIII which is a key molecule in the clotting cascade. Desmopressin acts to increase vWF and factor VIII concentration by encouraging its release from endothelial cell storage sites. Desmopressin is efficacious in type I and most type II disease but not in type III. This woman is known to have ‘mild’ disease thus making desmopressin a viable option. Interestingly, desmopressin in patients with type 2B will lead to a transient worsening of their thrombocytopenia. Patients with type 2B vWD have increased binding of the abnormal vWF to platelets causing sequestration and clearance of platelets. This is worsened for desmopressin, if only for a few hours. Despite this, there have been reports of patients benefiting from desmopressin.

Cryoprecipitate (A) is produced from freezing fresh plasma and thawing it at 4°C. The precipitate left is centrifuged and contains high concentrations of factor VIII, fibrinogen, fibronectin, factor XIII and vWF. It is not recommended in this patient as she has mild disease and the risk of viral transmission from blood products, although low, is not therefore justified.

Fresh frozen plasma (C) is prepared from whole blood or plasma collected by special techniques which is frozen with 8 hours of acquisition. It contains all of the coagulation factors and proteins present in the original blood. It is generally an overused resource but is useful for when complicated clotting issues need to be corrected quickly, e.g warfarin overdose, liver disease, DIC.

Vitamin K (D) is used to reverse warfarin overtreatment or in decompensated liver disease. Vitamin K is used for the synthesis of clotting factors II, VII, IX and X. It does not increase vWF which is required in this patient.

Recombinant factor VIII (E) is used in life-threatening surgery or emergency surgery in patients with haemophilia type A, not in mild vWD. If factor replacement were contemplated in patients with severe vWD (usually type III), then recombinant vWF would be used instead.

54
Q
  1. Myelodysplastic syndrome
    An 80-year-old man presents with tiredness and lethargy. After initial work-up, a diagnosis of myelodysplastic syndrome is suspected. Which of the following is true about this condition?

A A blood film will typically show neutrophil toxic granulation
B If there are 1 per cent blasts of the total white cell count, this represents leukaemic transformation
C Cytotoxic chemotherapy is first line treatment
D Mortality is more likely to be from infection than leukaemic transformation
E Absence of the short arm of chromosome 5 is a subtype

A

D
The myelodysplastic syndromes are a heterogeneous group of conditions characterized by an abnormal clone of stem cells with impaired prolif- eration and differentiation. The result is a peripheral cytopenia, qualita- tive abnormalities in erythroid, myeloid and megakaryocyte maturation, as well as increased risk of leukaemic transformation. The abnormalities, both quantitative and qualitative, in neutrophils mean susceptibility to bacterial infection is high and thus a corresponding increased likelihood of mortality (D). Skin infections are particularly common and resistant to treatment.

The peripheral blood film would not show toxic granulation (A) unless there is concomitant infection. ‘Toxic granulation’ is a term referring to the appearance of dark blue, coarse granules representing azurophilic granules with abnormal staining. Dohle bodies are light blue in colour and are found in the periphery of neutrophils in patients with infections but also myelodysplasia. They represent areas of rough endoplasmic reticulum with bound ribosomes. Another characteristic neutrophil change seen in MDS is the Pelger–Huet abnormality where there is a reduction in lobule number.

A blast content of 1% would not be considered as a leukaemic transformation. Refractory anaemia with an excess of blasts (RAEB) describes a type of MDS where there is an increased blast load implying progression to leukaemic transformation. Type I indicates a blast percentage between 5 and 10, whereas type II has a blast load of 10–19%. 20% blasts or more is therefore considered leukaemic transformation.

Absence of the long arm of chromosome 5 is a specific subtype of MDS. There is an interstitial deletion and it most commonly affects elderly women. It may respond quite dramatically to lenolidamide, a thalidomide derivative. Deletion of the short arm of chromosome 5 is associated with Cri du chat syndrome – a genetic disorder unrelated to MDS, named due to the characteristic cat cry made by affected children (‘cri du chat’ from French meaning ‘call of the cat’).

Cytotoxic therapy (C) is not usually used first line unless they are symptomatic or high risk using the International Prognostic Scoring system. Most treatment is supportive with the use of blood products, antibiotics to treat infection and growth factors such as eryhropoeitin or granulocyte-colony stimulating factor.

55
Q
  1. Bleeding disorders
A Immune thrombocytopenic purpura
B Idiopathic thrombotic thrombocytopenic purpura
C Disseminated intravascular coagulation
D Glanzmann’s thrombasthenia
E Von Willebrand disease 
F Haemophilia A
G Haemophilia B
H Hereditary haemorrhagic telangiectasia
I Bernard–Soulier syndrome

1 A 4-year-old girl is seen by her GP due to recent onset petechiae on her feet and bleeding of her gums when she brushes her teeth. The child’s platelet count is found to be 12 500 per μL. The GP prescribes prednisolone and reassures the child’s mother that the bleeding will resolve.

A

1) A

Immune thrombocytopenic purpura (ITP; A) may follow either an acute or chronic disease process. Acute ITP most commonly occurs in children, usually occurring 2 weeks after a viral illness. It is a type 2 hypersensitivity reaction, with IgG binding to virus-coated platelets. The fall in platelets is very low (less than 20×109/L) but is a self-limiting condition (few weeks). Chronic ITP is gradual in onset with no history of previous viral infection. It is also a type 2 hypersensitivity reaction with IgG targeting GLP-2b/3a.

Haemophilia B (G) is an X-linked genetic disorder characterized by a deficiency in factor 9. This can lead to bleeding spontaneously or in response to mild trauma.

Hereditary haemorrhagic telangiectasia (Osler–Weber–Rendu syndrome; H) is an autosomal dominant condition characterized by telangiectasia formation on the skin and mucous membranes leading to nose and gas- trointestinal bleeds.

Bernard–Soulier syndrome (I) is caused by a mutation of the glycoprotein GLP-1b, the receptor for von Willebrand factor in clot formation.

56
Q
  1. Bleeding disorders
A Immune thrombocytopenic purpura
B Idiopathic thrombotic thrombocytopenic purpura
C Disseminated intravascular coagulation
D Glanzmann’s thrombasthenia
E Von Willebrand disease 
F Haemophilia A
G Haemophilia B
H Hereditary haemorrhagic telangiectasia
I Bernard–Soulier syndrome

2 A 28-year-old man attends the haematology outpatient clinic regarding a long- standing condition he has suffered from. His disorder is related to a deficiency in factor 8 and therefore requires regular transfusions to replace this clotting factor.

A

2) F

Haemophilia A (F) is an X-linked genetic disorder and hence only affects men. Haemophilia A is characterized by a deficiency in factor 8. Haemophilia A is diagnosed by a reduced APTT as well as reduced factor 8. Symptoms depend on severity of disease: mild disease features bleeding after surgery/trauma; moderate disease results in bleeding after minor trauma; severe disease causes frequent spontaneous bleeds. Clinical features include haemarthrosis (causing fixed joints) and muscle haematoma (causing atrophy and short tendons).

Haemophilia B (G) is an X-linked genetic disorder characterized by a deficiency in factor 9. This can lead to bleeding spontaneously or in response to mild trauma.

Hereditary haemorrhagic telangiectasia (Osler–Weber–Rendu syndrome; H) is an autosomal dominant condition characterized by telangiectasia formation on the skin and mucous membranes leading to nose and gas- trointestinal bleeds.

Bernard–Soulier syndrome (I) is caused by a mutation of the glycoprotein GLP-1b, the receptor for von Willebrand factor in clot formation.

57
Q
  1. Bleeding disorders
A Immune thrombocytopenic purpura
B Idiopathic thrombotic thrombocytopenic purpura
C Disseminated intravascular coagulation
D Glanzmann’s thrombasthenia
E Von Willebrand disease 
F Haemophilia A
G Haemophilia B
H Hereditary haemorrhagic telangiectasia
I Bernard–Soulier syndrome

3 A 34-year-old man is taken to the local accident and emergency after suffering an episode of jaundice, fever and worsening headache. Blood tests reveal a low platelet count and blood film is suggestive of a microangiopathic haemolytic anaemia picture.

A

3) B

Idiopathic thrombotic thrombocytopenic purpura (B) occurs due to platelet microthrombi. Presenting features include microangiopathic haemolytic anaemia (red blood cells coming into contact with micro- scopic clots are damaged by shear stress), renal failure, thrombocyto- penia, fever and neurological signs (hallucinations/stroke/headache). A mutation in the ADAM-ST13 gene, coding for a protease that cleaves von Willebrand factor (vWF) allows for the formation of vWF multimers enabling platelet thrombi to form causing organ damage.

Haemophilia B (G) is an X-linked genetic disorder characterized by a deficiency in factor 9. This can lead to bleeding spontaneously or in response to mild trauma.

Hereditary haemorrhagic telangiectasia (Osler–Weber–Rendu syndrome; H) is an autosomal dominant condition characterized by telangiectasia formation on the skin and mucous membranes leading to nose and gas- trointestinal bleeds.

Bernard–Soulier syndrome (I) is caused by a mutation of the glycoprotein GLP-1b, the receptor for von Willebrand factor in clot formation.

58
Q
  1. Bleeding disorders
A Immune thrombocytopenic purpura
B Idiopathic thrombotic thrombocytopenic purpura
C Disseminated intravascular coagulation
D Glanzmann’s thrombasthenia
E Von Willebrand disease 
F Haemophilia A
G Haemophilia B
H Hereditary haemorrhagic telangiectasia
I Bernard–Soulier syndrome

4 A 68-year-old man on the Care of the Elderly ward is confirmed to have Gram- negative sepsis. The patient is bleeding from his mouth and is in shock. Initial blood tests reveal a reduced platelet count, anaemia and renal failure.

A

4) C

Disseminated intravascular coagulation (DIC; C) may be caused by Gram-negative sepsis, malignancy, trauma, placental abruption or amniotic fluid embolus. Tissue factor is released which triggers the activation of the clotting cascade, leading to platelet activation (thrombosis in microcirculation) and fibrin deposition (haemolysis). The consumption of platelets and clotting factors predisposes to bleeding. Plasmin is also generated in DIC which causes fibrinolysis, perpetuating the bleeding risk. The clinical manifestations of DIC are therefore linked to microthombus production (renal failure and neurological signs) and reduced platelets, clotting factors and increased fibrinolysis (bruising, gastrointestinal bleeding and shock).

Haemophilia B (G) is an X-linked genetic disorder characterized by a deficiency in factor 9. This can lead to bleeding spontaneously or in response to mild trauma.

Hereditary haemorrhagic telangiectasia (Osler–Weber–Rendu syndrome; H) is an autosomal dominant condition characterized by telangiectasia formation on the skin and mucous membranes leading to nose and gas- trointestinal bleeds.

Bernard–Soulier syndrome (I) is caused by a mutation of the glycoprotein GLP-1b, the receptor for von Willebrand factor in clot formation.

59
Q
  1. Bleeding disorders
A Immune thrombocytopenic purpura
B Idiopathic thrombotic thrombocytopenic purpura
C Disseminated intravascular coagulation
D Glanzmann’s thrombasthenia
E Von Willebrand disease 
F Haemophilia A
G Haemophilia B
H Hereditary haemorrhagic telangiectasia
I Bernard–Soulier syndrome

5 A 2-year-old boy is taken to see the GP due to his mother noticing bruising on his arms and legs after playing in the park. The parent mentions that she has also noticed several recent nose bleeds in her son but thought he would ‘grow out of it’. Investigations reveal a low APTT, low factor 8 levels and low Ristocetein cofactor activity.

A

5) E

von Willebrand disease (vWD; E) is an autosomal dominant condition caused by a mutation on chromosome 12. Physiologically, von Willebrand factor (vWF) has two roles: platelet adhesion and factor 8 production. Therefore, in vWD, where there is a deficiency in vWF, there is a defect in platelet plug formation as well as low levels of fac- tor 8. Clinically, patients will present with gum bleeding, epistaxis or prolonged bleeding after surgery. Investigations will reveal a high/ normal APTT, low factor 8 levels, low ristocetin cofactor activity, poor ristocetin aggregation and normal PTT, Glanzmann’s thrombasthenia (D) is caused by a mutation of GLP-2b/3a, a glycoprotein that is essential for platelet aggregation, and hence blood coagulation.

Haemophilia B (G) is an X-linked genetic disorder characterized by a deficiency in factor 9. This can lead to bleeding spontaneously or in response to mild trauma.

Hereditary haemorrhagic telangiectasia (Osler–Weber–Rendu syndrome; H) is an autosomal dominant condition characterized by telangiectasia formation on the skin and mucous membranes leading to nose and gas- trointestinal bleeds.

Bernard–Soulier syndrome (I) is caused by a mutation of the glycoprotein GLP-1b, the receptor for von Willebrand factor in clot formation.

60
Q
4. Haemolytic anaemia (1)
An 8-year-old boy is brought to his GP by his father, who reports that he has been feeling progressively more tired over the past few months. On examination the GP notices a slight yellowing of his sclera, and the presence of splenomegaly. His father recollects that he himself was told he had a problem with his blood cells as a child, but has never been affected by it. A peripheral blood film shows a raised reticulocyte count and spherocytes. He is likely to have a positive:
A Coombs test
B Osmotic fragility test
C G6PD test
D Sickle cell screen
E Schilling test
A

B
Hereditary spherocytosis is a type of autosomal dominant inherited haemolytic anaemia. It occurs due to an increase in the fragility of
the red blood cell membrane due to dysfunctional skeletal proteins in the membrane, such as spectrin, ankyrin and band 4.2. Most patients develop a haemolytic state that is partially compensated. Clinical features can include tiredness from anaemia, as in this case, and the presence of jaundice and splenomegaly on examination. They can also develop pigment gallstones from the haemolysis. As with this child, there is often a positive family history.
A blood film can show the presence of spherocytes and reticulocytes, and a Coombs test is negative. They may have a positive osmotic fragil- ity test (B), but remember that this is just used to confirm that there are spherocytes present, not that the cause is hereditary spherocytosis. With this test, because the membrane is more permeable to salt and water, the spherocytes rupture in a mildly hypotonic solution. Do not forget that spherocytes may also be found in autoimmune haemolytic anaemia.
To go back to basics, remember that common laboratory features of all haemolytic anaemias might include:
- anaemia
- reticulocytosis
- raised bilirubin (unconjugated)
- raised LDH
- reduced haptoglobins (a plasma protein that binds free haemoglobin)
It is then worth classifying haemolytic anaemias as inherited or acquired to help you remember the different types. Hereditary anaemias can be thought of as due to defects in the red cell, such as the mem- brane (such as in spherocytosis and elliptocytosis), the haemoglobin itself (structural defects in sickle cell disease (D) or quantitative defects in thalassaemias), or of the enzymes inside the cell (such as in glucose- 6-phosphate deficiency (C)).
Acquired haemolytic anaemias can be immune or non-immune. Immune haemolytic anaemias can include autoimmune haemolytic anaemia, which might result in the formation of spherocytes and is direct anti- globulin test (DAT) or Coombs test positive (A). In the Coombs test, red blood cells are washed and incubated with Coombs reagent (anti-human globulin). In a positive test, this produces agglutination of the red blood cells (RBCs) which indicates that antibodies or complement proteins have become bound to the red blood cell membrane. So spherocytes
can occur both in hereditary spherocytosis and in AIHA, with the latter being Coombs test positive.
Other immune haemolytic anaemias can occur in the presence of underlying autoimmune disease or lymphomas. Non-immune acquired haemolytic anaemias can occur due to infections such as malaria, or microangiopathic haemolytic anaemia.
The Schiling test (E) is used for vitamin B12 deficiency to determine if the cause is pernicious anaemia.

61
Q
  1. Factor V Leiden

A middle-aged woman comes to the dermatology clinic with a suspicious mole on her back. You decide excision is required and during the history she says ‘I have Factor V Leiden’. Which of the following best describes the pathophysiology of Factor V Leiden mutation?

A Prothrombin mutation
B Activated protein C resistance
C Antithrombin III deficiency
D Protein C deficiency
E Protein S deficiency
A

B
Factor V Leiden is an autosomal dominantly inherited point mutation where arginine is replaced by glutamine in the 506th position. Factor V normally circulates in plasma as an inactivated factor and is activated by thrombin which then acts as a co-factor, with factor Xa, to convert prothrombin to thrombin. Factor Va is inactivated by cleavage of its heavy chain; firstly at position Arg506, which causes conformational change to reveal a further two cleavage sites (Arg306 and Arg 679). This inactivation is performed by the activated protein C complex, and thus the Leiden mutation confers resistance (B). The prothrombotic consequence of Factor V Leiden is actually two-fold – first Factor V is degraded more slowly thus there is more generation of thrombin in the prothrombinase complex and second, once factor V is cleaved at the first Arg506 site, it is thought to play a role, with protein S, to support activated protein C in Factor VIIIa degradation too.

Prothrombin mutation (A) where there is a guanine to adenine transi- tion at the 20210 in the 3 ́ untranslated region of the prothrombin gene is related to factor V Leiden. It is, however, another cause of inherited hypercoagulability. It is thought the mutation may alter the efficiency of mRNA processing or increase the prothrombin mRNA half life.

Antithrombin III deficiency (B) is caused by a mutation in the long arm of chromosome 1 where a lack of antithrombin III leads to a hyper- coaguable state. This is because antithrombin III is an important anti- coagulant acting to inhibit thrombin and factors Xa and IXa. Its activity is increased 4000-fold by heparin.

Protein C (D) and Protein S (E) deficiency are other types of inherited hypercoagulable states not related to Factor V Leiden. Protein C is the key component in the activated protein C complex which degrades factors Va and VIIIa. The activated protein C complex effect is markedly increased by the presence of protein S, hence its importance in anticoagulation.

62
Q
5. Haemolytic anaemia (2)
A 33-year-old Turkish man presents with extreme tiredness and shortness of breath after being started on a course of anti-malarial tablets. A full blood count reveals an Hb of 6.8. His Coombs test is negative. The cell type most likely to be found on his blood film is:
A Heinz bodies
B Pencil cells
C Target cells
D Spherocytes
E Sickle cells
A
A This man is suffering from glucose-6-phosphate dehydrogenase (G6PD) deficiency, an X-linked recessive disorder that is common in peo-
ple from the Mediterranean, South East Asia, Middle East and West Africa. This enzyme is responsible for maintaining levels of glutathione from the pentose phosphate pathway, which protects against oxidant free radicals. Oxidative stress, for example in the form of chemicals, food or infection, can put people with this condition at risk of severe haemolytic anaemia. Drugs to be avoided in these patients include anti-malarials, such as primaquine, and others such as sulphonamides, vitamin K and dapsone. The exam favourite of broad beans can lead to a reaction called favism in these patients.
Heinz bodies (A) are characteristically found on the blood film during
a crisis: these are small inclusions within the red cell due to denatured haemoglobin. Remember that the blood count can actually be normal in between crises. The level of G6PD itself can also be assayed, but can be falsely negative during active haemolysis so may be delayed until a few weeks after an acute episode.
Pencil cells (B) are a type of elliptocyte that occur in iron deficiency anaemia, thalassaemia and pyruvate kinase deficiency. Target cells (C) have a central dense area with a ring of pallor, and can occur in the three Hs: hepatic pathology, hyposplenism and haemoglobinopathies. Spherocytes (D) are found in hereditary spherocytosis, where an increase in the fragility of the red blood cell membrane occurs due to dysfunc- tional skeletal proteins in the membrane, such as spectrin, ankyrin and band 4.2. They can also be found in haemolytic anaemia. Sickle cells (E) are found in sickle cell anaemia, but not in sickle cell trait.
63
Q
  1. Thrombophilia
A Factor V Leiden
B Antiphospholipid syndrome
C Malignancy
D Protein S deficiency
E Antithrombin deficiency
F Prothrombin G20210A mutation 
G Oral contraceptive pill
H Buerger’s disease
I Chronic liver diseas

1 A 35-year-old Caucasian man presents to accident and emergency with deep pain and swelling in his left calf. His past medical history reveals history of recurrent DVTs. The patient’s notes reveal a letter from his haematologist who had diagnosed a condition caused by a substitution mutation.

A

1) F

Prothrombin G20210A (F) is an inherited thrombophilia caused by the substitution of guanine with adenine at the 20210 position of the prothrombin gene. Physiologically, prothrombin promotes clotting after a blood vessel has been damaged. The G20210A causes the amplification of prothrombin production thereby increasing the risk of clotting, and causing a predisposition to deep vein thrombosis and pulmonary embolism. The prevalence of the mutation is approximately 5 per cent in the Caucasian population, the race with the greatest preponderance.

Malignancy (C) may predispose to thrombosis due to venous thrombosis. Some tumour cells also express tissue factor, most notably pancreatic cancer cells.

Antithrombin deficiency (E) is an inherited cause of venous thrombosis. Under physiological conditions antithrombin inhibits clotting factors thrombin and factor 10a.

The oral contraceptive pill (G) may cause venous thrombosis secondary to increased circulating oestrogens, amplifying the synthesis of clotting factors by the liver.

Chronic liver disease (I) results in reduced clotting factor production by the liver as well as abnormalities of platelet function.

64
Q
  1. Thrombophilia
A Factor V Leiden
B Antiphospholipid syndrome
C Malignancy
D Protein S deficiency
E Antithrombin deficiency
F Prothrombin G20210A mutation 
G Oral contraceptive pill
H Buerger’s disease
I Chronic liver diseas

2 A 38-year-old woman presents to accident and emergency with abdominal pain as well as passing blood and tissue per vagina. Ectopic pregnancy is diagnosed after ultrasound. The patient’s past medical history includes a haematological condition in which a clotting factor is unable to be degraded by activated protein C.

A

2) A

Factor V Leiden (A) is an autosomal dominant inherited thrombophilia. Under normal circumstances protein C inhibits factor 5. In Factor V Leiden a mutation of the F5 gene that codes for factor 5, whereby an arginine codon is substituted for a glutamine codon, results in impaired degradation of factor 5 by protein C. As a result, patients are at risk of deep vein thrombosis and miscarriage. Diagnostic tests determine the functionality of activated protein C.

Malignancy (C) may predispose to thrombosis due to venous thrombosis. Some tumour cells also express tissue factor, most notably pancreatic cancer cells.

Antithrombin deficiency (E) is an inherited cause of venous thrombosis. Under physiological conditions antithrombin inhibits clotting factors thrombin and factor 10a.

The oral contraceptive pill (G) may cause venous thrombosis secondary to increased circulating oestrogens, amplifying the synthesis of clotting factors by the liver.

Chronic liver disease (I) results in reduced clotting factor production by the liver as well as abnormalities of platelet function.

65
Q
  1. Thrombophilia
A Factor V Leiden
B Antiphospholipid syndrome
C Malignancy
D Protein S deficiency
E Antithrombin deficiency
F Prothrombin G20210A mutation 
G Oral contraceptive pill
H Buerger’s disease
I Chronic liver diseas

3 A 32-year-old woman is seen by her rheumatologist to follow up her long- standing systemic lupus erythematosus (SLE). The patient has a history of recurrent miscarriages. The woman is positive for anti-cardiolipin antibodies and lupus anticoagulant.

A

3) B

Antiphospholipid syndrome (APLS; B) is an autoimmune disorder that may present with stroke (arterial thrombosis), deep vein thrombosis (venous thrombosis) and/or recurrent miscarriages. APLS may be primary (not associated with autoimmune disease) or secondary to auto-immune disease such as SLE. Anti-cardiolipin antibodies and lupus anticoagulant bind to phospholipids on cell surface membranes of cells causing the activation of the coagulation cascade and thereby promoting clot formation. Diagnosis involves demonstrating the presence of circulating anti-cardiolipin antibodies and lupus anticoagulant.

Malignancy (C) may predispose to thrombosis due to venous thrombosis. Some tumour cells also express tissue factor, most notably pancreatic cancer cells.

Antithrombin deficiency (E) is an inherited cause of venous thrombosis. Under physiological conditions antithrombin inhibits clotting factors thrombin and factor 10a.

The oral contraceptive pill (G) may cause venous thrombosis secondary to increased circulating oestrogens, amplifying the synthesis of clotting factors by the liver.

Chronic liver disease (I) results in reduced clotting factor production by the liver as well as abnormalities of platelet function.

66
Q
  1. Thrombophilia
A Factor V Leiden
B Antiphospholipid syndrome
C Malignancy
D Protein S deficiency
E Antithrombin deficiency
F Prothrombin G20210A mutation 
G Oral contraceptive pill
H Buerger’s disease
I Chronic liver diseas

4 A 45-year-old man, who has a 50 pack/year history of smoking, is referred to the vascular outpatient clinic by his GP after suffering intermittent claudication. A diagnostic angiogram reveals a corkscrew appearance of his lower limb arteries.

A

4) H

Buerger’s disease (thromboangitis obliterans; H) is a vasculitis of small/ medium arteries and veins of the hands and feet; it is strongly related to smoking. Claudication may be the initial presentation but as the disease progresses there is an association with recurrent arterial and venous thrombosis leading to gangrene and amputation in severe cases. Angiograms of the upper and lower limbs are helpful in the diagnosis of Buerger’s disease; a corkscrew appearance of the arteries may arise due to persistent vascular damage.

Malignancy (C) may predispose to thrombosis due to venous thrombosis. Some tumour cells also express tissue factor, most notably pancreatic cancer cells.

Antithrombin deficiency (E) is an inherited cause of venous thrombosis. Under physiological conditions antithrombin inhibits clotting factors thrombin and factor 10a.

The oral contraceptive pill (G) may cause venous thrombosis secondary to increased circulating oestrogens, amplifying the synthesis of clotting factors by the liver.

Chronic liver disease (I) results in reduced clotting factor production by the liver as well as abnormalities of platelet function.

67
Q
  1. Thrombophilia
A Factor V Leiden
B Antiphospholipid syndrome
C Malignancy
D Protein S deficiency
E Antithrombin deficiency
F Prothrombin G20210A mutation 
G Oral contraceptive pill
H Buerger’s disease
I Chronic liver diseas

5 A 37-year-old man presents to accident and emergency with shortness of breath and severe pleuritic chest pain. A CTPA reveals the diagnosis of pulmonary embolism. The patient’s haematological records state the patient has a condition that leads to the persistence of factors 5a and 8a causing increased risk of venous thrombosis.

A

5) D

Protein S deficiency (D) is associated with the impaired degradation of factors Va and VIIIa. Protein S and protein C are physiological anti- coagulants. Deficiency of protein S leads to persistence of factors 5a and 8a in the circulation and hence patients have a susceptibility to venous thrombosis. Three types of protein S deficiency exist: type I (quantitative defect) and types II and III (qualitative defect). Since protein S is a vitamin K dependent anticoagulant, warfarin treatment and liver disease may also lead to venous thrombosis in rare cases (the majority of cases show increased bleeding).

Malignancy (C) may predispose to thrombosis due to venous thrombosis. Some tumour cells also express tissue factor, most notably pancreatic cancer cells.

Antithrombin deficiency (E) is an inherited cause of venous thrombosis. Under physiological conditions antithrombin inhibits clotting factors thrombin and factor 10a.

The oral contraceptive pill (G) may cause venous thrombosis secondary to increased circulating oestrogens, amplifying the synthesis of clotting factors by the liver.

Chronic liver disease (I) results in reduced clotting factor production by the liver as well as abnormalities of platelet function.

68
Q
  1. Complications of transfusion
A Immediate haemolytic transfusion reaction
B Febrile non-haemolytic reaction
C Iron overload
D IgA deficiency
E Transfusion related lung injury
F Bacterial infection
G Delayed haemolytic transfusion reaction
H Fluid overload
I Graft versus host disease

1 An 82-year-old man has just received a blood transfusion following a low haemoglobin level on the Care of the Elderly ward. He is now short of breath and is coughing up pink frothy sputum.

A

1) H

Fluid overload (H) is an immediate complication of blood transfusion. Clinical features suggestive of fluid overload will include dyspnoea, distended neck veins and pink frothy sputum. Usually fluid overload occurs in situations where the blood transfusion rate is too fast; a transfusion would generally have to run at more than 2mL/kg/hour to induce fluid overload. Patients with pre-existing cardiac or renal failure are prone to fluid overload as a result of blood transfusion.

Febrile non-haemolytic reaction (B) occurs after pregnancy when anti- leukocytic antibodies can form; this causes a reaction to leukocytes in subsequent transfusions. Low fever and rigors are characteristic.

Iron overload (C) may occur in patients who have regular blood transfusions for conditions such as thalassaemia or sickle cell disease. Features include a bronzed discolouration to the skin, short stature and heart failure.

Bacterial infection (F) caused by blood transfusion is categorized by high fevers, rigors and hypotension. Organisms that can be transmitted if the blood is not screened include hepatitis B, hepatitis C and HIV.

Delayed haemolytic transfusion reaction (G), also known as non-ABO transfusion reaction, occurs more than 24 hours after the transfusion. Clinical effects are milder than immediate haemolytic transfusion reaction.

69
Q
  1. Complications of transfusion
A Immediate haemolytic transfusion reaction
B Febrile non-haemolytic reaction
C Iron overload
D IgA deficiency
E Transfusion related lung injury
F Bacterial infection
G Delayed haemolytic transfusion reaction
H Fluid overload
I Graft versus host disease

2 A 34-year-old HIV-positive man receives a regular blood transfusion as part of his beta-thalassaemia major treatment regimen. He soon develops diarrhoea and a maculopapular rash on his limbs.

A

2) I

Graft versus host disease (GVHD; I) occurs due to the transfer of donor lymphocytes to the recipient in a blood transfusion in patients who are immunosuppressed. Normally, the immune system is strong enough to detect and destroy donor lymphocytes. However, in immunosuppression (stem cell transplant patients/chemotherapy/malignancy/HIV) the donor lymphocytes cannot be destroyed; these foreign lymphocytes persist and target host tissue, especially the gastrointestinal tract and skin. Symptoms of GVHD include diarrhoea, maculopapular rash and skin necrosis. To minimize GVHD, donor blood is irradiated to remove lymphocytes.

Febrile non-haemolytic reaction (B) occurs after pregnancy when anti- leukocytic antibodies can form; this causes a reaction to leukocytes in subsequent transfusions. Low fever and rigors are characteristic.

Iron overload (C) may occur in patients who have regular blood transfusions for conditions such as thalassaemia or sickle cell disease. Features include a bronzed discolouration to the skin, short stature and heart failure.

Bacterial infection (F) caused by blood transfusion is categorized by high fevers, rigors and hypotension. Organisms that can be transmitted if the blood is not screened include hepatitis B, hepatitis C and HIV.

Delayed haemolytic transfusion reaction (G), also known as non-ABO transfusion reaction, occurs more than 24 hours after the transfusion. Clinical effects are milder than immediate haemolytic transfusion reaction.

70
Q
  1. Complications of transfusion
A Immediate haemolytic transfusion reaction
B Febrile non-haemolytic reaction
C Iron overload
D IgA deficiency
E Transfusion related lung injury
F Bacterial infection
G Delayed haemolytic transfusion reaction
H Fluid overload
I Graft versus host disease

3 A 34-year-old man requires a blood transfusion following a road traffic accident. However, soon after the transfusion, the patient is dyspnoeic and hypotensive. Investigation into the patient’s past medical history reveals a history of recurrent chest and gastrointestinal infections.

A

3) D

IgA deficiency (D) leads to recurrent mild infections of the mucous membranes lining the airways and digestive tract. In affected patients serum IgA levels are undetectable but IgG and IgM levels are normal. IgA is found in mucous secretions from the respiratory and gastrointestinal tracts and plays a key role in mucosal immunity. IgA deficient patients are also predisposed to severe anaphylactic reactions to blood transfusions due to the presence of IgA in donor blood.

Febrile non-haemolytic reaction (B) occurs after pregnancy when anti-leukocytic antibodies can form; this causes a reaction to leukocytes in subsequent transfusions. Low fever and rigors are characteristic.

Iron overload (C) may occur in patients who have regular blood transfusions for conditions such as thalassaemia or sickle cell disease. Features include a bronzed discolouration to the skin, short stature and heart failure.

Bacterial infection (F) caused by blood transfusion is categorized by high fevers, rigors and hypotension. Organisms that can be transmitted if the blood is not screened include hepatitis B, hepatitis C and HIV.

Delayed haemolytic transfusion reaction (G), also known as non-ABO transfusion reaction, occurs more than 24 hours after the transfusion. Clinical effects are milder than immediate haemolytic transfusion reaction.

71
Q
  1. Complications of transfusion
A Immediate haemolytic transfusion reaction
B Febrile non-haemolytic reaction
C Iron overload
D IgA deficiency
E Transfusion related lung injury
F Bacterial infection
G Delayed haemolytic transfusion reaction
H Fluid overload
I Graft versus host disease

4 A 56-year-old man is given a blood transfusion following severe blood loss after a hip replacement operation. Three hours after the transfusion, the patient develops shortness of breath, a dry cough and a fever of 39°C.

A

4) E

Transfusion-related lung injury (TRALI; E) is characterized by acute non-cardiogenic pulmonary oedema that occurs within 6 hours follow- ing blood transfusion. The pathogenesis of TRALI involves the presence of anti-white blood cell antibodies in the donor blood that attack host leukocytes; sensitizing events in donors include previous blood transfusion or transplantation. Clinical features of TRALI are dry cough, dyspnoea and fever.

Febrile non-haemolytic reaction (B) occurs after pregnancy when anti- leukocytic antibodies can form; this causes a reaction to leukocytes in subsequent transfusions. Low fever and rigors are characteristic.

Iron overload (C) may occur in patients who have regular blood transfusions for conditions such as thalassaemia or sickle cell disease. Features include a bronzed discolouration to the skin, short stature and heart failure.

Bacterial infection (F) caused by blood transfusion is categorized by high fevers, rigors and hypotension. Organisms that can be transmitted if the blood is not screened include hepatitis B, hepatitis C and HIV.

Delayed haemolytic transfusion reaction (G), also known as non-ABO transfusion reaction, occurs more than 24 hours after the transfusion. Clinical effects are milder than immediate haemolytic transfusion reaction.

72
Q
  1. Complications of transfusion
A Immediate haemolytic transfusion reaction
B Febrile non-haemolytic reaction
C Iron overload
D IgA deficiency
E Transfusion related lung injury
F Bacterial infection
G Delayed haemolytic transfusion reaction
H Fluid overload
I Graft versus host disease

5 A 29-year-old woman requires an immediate blood transfusion after suffering a post-partum haemorrhage. However, 30 minutes after her transfusion she develops abdominal pain, facial flushing and vomiting. Analysis of the woman’s urine reveals the presence of haemoglobin.

A

5) A

Immediate haemolytic transfusion reaction (IHTR; A) is characterized by ABO incompatibility and occurs 1–2 hours post-transfusion. Clinical features include abdominal pain, loin pain, facial flushing, vomiting and haemoglobinuria. Host IgG and IgM target donor red blood cells which are subsequently removed by the reticuloendothelial system. The most severe reaction occurs if a group O patient is transfused with group A blood.

Febrile non-haemolytic reaction (B) occurs after pregnancy when anti- leukocytic antibodies can form; this causes a reaction to leukocytes in subsequent transfusions. Low fever and rigors are characteristic.

Iron overload (C) may occur in patients who have regular blood transfusions for conditions such as thalassaemia or sickle cell disease. Features include a bronzed discolouration to the skin, short stature and heart failure.

Bacterial infection (F) caused by blood transfusion is categorized by high fevers, rigors and hypotension. Organisms that can be transmitted if the blood is not screened include hepatitis B, hepatitis C and HIV.

Delayed haemolytic transfusion reaction (G), also known as non-ABO transfusion reaction, occurs more than 24 hours after the transfusion. Clinical effects are milder than immediate haemolytic transfusion reaction.

73
Q
6. Infectious mononucleosis
A 25-year-old student is treated for infectious mononucleosis following a posi- tive Paul Bunnell test. A blood film reveals target cells, Howell–Jolly bodies and atypical lymphocytes. Together, these suggest that he has features of:
A Bone marrow suppression
B Hyposplenism
C Disseminated intravascular coagulation
D Haemolytic anaemia
E Liver failure
A

B

Up to half of all patients might develop splenomegaly in infectious mononucleosis. This does not often cause symptoms but can lead to splenic rupture, either spontaneously or following minor trauma, and may necessitate treatment with splenectomy. Postoperatively a combination of features on a blood film might suggest hyposplenism:

Howell–Jolly bodies: these are small fragments of non-functional nuclei that are normally removed by the spleen, so might be seen on a blood film in hyposplenism. They may also be seen in megaloblastic and IDAs

Target cells: these have a central dense area with a ring of pallor, and can occur in the three Hs: hepatic pathology, hyposplenism and haemoglobinopathies
Occasional nucleated red blood cells
Lymphocytosis
Macrocytosis

Acanthocytes: spiculated red cells that are found in hyposplenism, alpha-beta-lipoproteinaemia, chronic liver disease and alpha-thalassaemia trait

Atypical lymphocytes are large lymphocytes which vary in size and shape, and might be seen in infectious mononucleosis (even if the patient has not developed hyposplenism).

Other causes of hyposplenism:

1) Traumatic, i.e. following an accident or during surgery
2) Planned splenectomy – prophylactically in massive splenomegaly or hypersplenism (e.g. hereditary spherocytosis or elliptocytosis)
3) Physiological hyposplenism, e.g. in sickle cell anaemia, coeliac disease, or ulcerative colitis

To remember in which haematological diseases a splenectomy may provide a substantial benefit, the following mnemonic may be useful: The PIIES = Thalassaemia, Pyruvate kinase deficiency, Immune haemolytic anaemia, Idiopathic thrombocytopenic purpura, Elliptocytosis, and Spherocytosis (hereditary).

Importantly, splenectomy patients are at increased risk of sepsis from capsulated organisms. They therefore require lifelong:
1 Penicillin V prophylaxis
2 Pneumococcal conjugate vaccine
3 Human influenza b (Hib) vaccine
4 Meningococcal vaccine
74
Q
  1. Haematological neoplasms (1)
A Acute lymphoblastic leukaemia
B Acute promyelocytic leukaemia
C Chronic myeloid leukaemia
D Chronic lymphocytic leukaemia
E Hairy cell leukaemia
F T-cell prolymphocytic leukaemia 
G Large granular lymphocytic leukaemia
H Adult T-cell leukaemia
I Acute myeloid leukaemia

1 A 62-year-old woman is seen by a GP due to a recent chest infection that has been troubling her. Initial blood tests show an elevated white cell count with specifically raised granulocytes. Following referral to a haematologist, a bone marrow biopsy reveals a hypercellular bone marrow and cytogenetic screening suggests a translocation between chromosomes 9 and 22.

A

1) C

Chronic myeloid leukaemia (CML; C) is most prevalent in the elderly population and is commonly suspected secondary to routine blood tests. Blood results will show an elevated granulocytes (neutrophils, basophils and eosinophils). Blood film will demonstrate myeloid cells at different stages of maturation. Bone marrow biopsy in CML patients suggests hypercellularity. 95% of cases are caused by the Philadelphia chromosome, a chromosomal translocation between chromosomes 9 and 22; this results in the BCR-Abl fusion oncogene that has tyrosine kinase activity. Recent novel therapies for CML include imatinib, a BCR-Abl inhibitor.

Acute lymphoblastic leukaemia (ALL; A) is the most common paedi- atric cancer, characterized by the presence of >20% lymphoblasts in the bone marrow due to suppressed maturation and uncontrolled proliferation.

Acute promyelocytic leukaemia (APML; B) is the M3 subtype of acute myeloid leukaemia. It is caused by a translocation mutation forming PML-RAR leading to proliferation of promyelocytes.

T-cell prolymphocytic leukaemia (T-PLL; F) is an aggressive T-cell leukaemia. The most common causative mutation is an inversion in chromosome 14: inv 14(q11;q32).

Large granular lymphocytic leukaemia (G) is characterized by the presence of large lymphocytes in the blood stream and bone marrow that contain azurophilic granules.

75
Q
  1. Haematological neoplasms (1)
A Acute lymphoblastic leukaemia
B Acute promyelocytic leukaemia
C Chronic myeloid leukaemia
D Chronic lymphocytic leukaemia
E Hairy cell leukaemia
F T-cell prolymphocytic leukaemia 
G Large granular lymphocytic leukaemia
H Adult T-cell leukaemia
I Acute myeloid leukaemia

2 A 41-year-old man is referred to a haematologist by his general practitioner after several recent chest infections and tiredness. On examination, bruises are seen on his lower limbs as well as splenomegaly. Initial blood tests reveal a pancytopenia. Further testing demonstrates the presence of tumour cells that express tartrate-resistant acid phosphatase.

A

2) E

Hairy cell leukaemia (HCL; E) is a haematological malignancy of B lymphocytes and a subtype of chronic lymphocytic leukaemia. It most commonly occurs in middle-aged men. The cancer derives its name from the fine hair-like projections that are seen on tumour cells on microscopy. Cell surface markers include CD25 (IL-2 receptor) and CD11c (adhesion molecule). Diagnosis can be confirmed by the presence of tartrate-resistant acid phosphatase (TRAP) on cytochemical analysis. Clinical features relate to invasion of the spleen (splenomegaly), liver (hepatomegaly) and bone marrow (pancytopenia).

Acute lymphoblastic leukaemia (ALL; A) is the most common paedi- atric cancer, characterized by the presence of >20% lymphoblasts in the bone marrow due to suppressed maturation and uncontrolled proliferation.

Acute promyelocytic leukaemia (APML; B) is the M3 subtype of acute myeloid leukaemia. It is caused by a translocation mutation forming PML-RAR leading to proliferation of promyelocytes.

T-cell prolymphocytic leukaemia (T-PLL; F) is an aggressive T-cell leukaemia. The most common causative mutation is an inversion in chromosome 14: inv 14(q11;q32).

Large granular lymphocytic leukaemia (G) is characterized by the presence of large lymphocytes in the blood stream and bone marrow that contain azurophilic granules.

76
Q
  1. Haematological neoplasms (1)
A Acute lymphoblastic leukaemia
B Acute promyelocytic leukaemia
C Chronic myeloid leukaemia
D Chronic lymphocytic leukaemia
E Hairy cell leukaemia
F T-cell prolymphocytic leukaemia 
G Large granular lymphocytic leukaemia
H Adult T-cell leukaemia
I Acute myeloid leukaemia

3 A 60-year-old man presents to his GP with fever, malaise and cough. On examination, the man is found to have petechiae on his legs as well as gum hypertrophy. Blood tests reveal anaemia, leukocytopenia and thrombocytopenia. A blood film demonstrates the presence of Auer rods within blast cells.

A

3) I

Acute myeloid leukaemia (AML; I) is characterized by more than 20% myleoblasts in the bone marrow. AML also causes proliferation of megakaryocytes and erythrocytes. Mutations that can cause AML include internal tandem duplications of the FLT3 gene (coding for a tyrosine kinase) and t(8;21) (a translocation causing a compressor complex to inhibit haematopoietic differentiation. Primary causes include Down’s syndrome; secondary causes include myeloproliferative disease. Blood tests will reveal a variable WCC, anaemia, thrombocytopenia and reduced neutrophil count. Auer rods on blood film are pathognomonic, which will also be leukoerythroblastic. Immunophenotyping of CD13, CD33 or CD34 can also aid diagnosis.

Acute lymphoblastic leukaemia (ALL; A) is the most common paedi- atric cancer, characterized by the presence of >20% lymphoblasts in the bone marrow due to suppressed maturation and uncontrolled proliferation.

Acute promyelocytic leukaemia (APML; B) is the M3 subtype of acute myeloid leukaemia. It is caused by a translocation mutation forming PML-RAR leading to proliferation of promyelocytes.

T-cell prolymphocytic leukaemia (T-PLL; F) is an aggressive T-cell leukaemia. The most common causative mutation is an inversion in chromosome 14: inv 14(q11;q32).

Large granular lymphocytic leukaemia (G) is characterized by the presence of large lymphocytes in the blood stream and bone marrow that contain azurophilic granules.

77
Q
  1. Haematological neoplasms (1)
A Acute lymphoblastic leukaemia
B Acute promyelocytic leukaemia
C Chronic myeloid leukaemia
D Chronic lymphocytic leukaemia
E Hairy cell leukaemia
F T-cell prolymphocytic leukaemia 
G Large granular lymphocytic leukaemia
H Adult T-cell leukaemia
I Acute myeloid leukaemia

4 A 42-year-old Japanese migrant presents to his GP with generalized lymphadenopathy and nodules on his arms. On examination the patient has hepatosplenomegaly. Blood tests reveal lymphocytosis and a raised calcium level.

A

4) H

Adult T-cell leukaemia (adult T-cell lymphoma; ATL; H) is a rare haematological malignancy with poor prognosis. It is caused by human T-cell leukaemia virus type 1 (HTLV-1), endemic in Japan and the Caribbean. Tumour cells express the cell surface protein CD4 and will contain the HTLV-1 virus within; the nuclei of ATL cells have a characteristic cloverleaf appearance. Clinical features include lymphadenopathy, hepatosplenomegaly, skin lesions and hypercalcaemia.

Acute lymphoblastic leukaemia (ALL; A) is the most common paedi- atric cancer, characterized by the presence of >20% lymphoblasts in the bone marrow due to suppressed maturation and uncontrolled proliferation.

Acute promyelocytic leukaemia (APML; B) is the M3 subtype of acute myeloid leukaemia. It is caused by a translocation mutation forming PML-RAR leading to proliferation of promyelocytes.

T-cell prolymphocytic leukaemia (T-PLL; F) is an aggressive T-cell leukaemia. The most common causative mutation is an inversion in chromosome 14: inv 14(q11;q32).

Large granular lymphocytic leukaemia (G) is characterized by the presence of large lymphocytes in the blood stream and bone marrow that contain azurophilic granules.

78
Q
  1. Haematological neoplasms (1)
A Acute lymphoblastic leukaemia
B Acute promyelocytic leukaemia
C Chronic myeloid leukaemia
D Chronic lymphocytic leukaemia
E Hairy cell leukaemia
F T-cell prolymphocytic leukaemia 
G Large granular lymphocytic leukaemia
H Adult T-cell leukaemia
I Acute myeloid leukaemia

5 A 70-year-old man is reviewed by his GP after having felt tired and experienced weight loss over a 2-month period. The patient has lymphadenopathy on examination. Blood tests demonstrates a lymphocytosis of 4500 cells per microlitre and smudge cells can be visualized on a peripheral blood film.

A

5) D

Chronic lymphocytic leukaemia (CLL; D) is a B-cell neoplasm characterized by a lymphocyte count of over 4000 cells per microlitre. CLL most commonly occurs in elderly men. The cancer presents primarily in the lymph nodes with small lymphocytes containing irregular nuclei mixed with larger prolymphocytes. Prolymphocytes may aggregate to form pathognomonic proliferation centres. Blood film may reveal the presence of smudge cells. Clinical features are non-specific (tiredness & weight loss). Hypogammaglobulinaemia is an associated immune phenomenon. CLL may convert into more aggressive forms including prolymphocytic transformation and diffuse-large B-cell lymphoma (Richter’s syndrome).

Acute lymphoblastic leukaemia (ALL; A) is the most common paediatric cancer, characterized by the presence of >20% lymphoblasts in the bone marrow due to suppressed maturation and uncontrolled proliferation.

Acute promyelocytic leukaemia (APML; B) is the M3 subtype of acute myeloid leukaemia. It is caused by a translocation mutation forming PML-RAR leading to proliferation of promyelocytes.

T-cell prolymphocytic leukaemia (T-PLL; F) is an aggressive T-cell leukaemia. The most common causative mutation is an inversion in chromosome 14: inv 14(q11;q32).

Large granular lymphocytic leukaemia (G) is characterized by the presence of large lymphocytes in the blood stream and bone marrow that contain azurophilic granules.

79
Q
7. Haemoglobinopathies in children (1)
A 4-year-old Afro-Caribbean boy has chest and abdominal pain. His blood tests reveal an Hb of 6.1g/dL, with an MCV of 65. A blood film shows the presence of sickle cells. The most likely diagnosis is:
A Sickle cell trait
B Sickle cell anaemia
C Sickle cell/ -thalassaemia
D Sickle cell/haemoglobin C
E Beta-Thalassaemia
A

B
This boy is suffering from sickle cell anaemia (B), an autosomal recessive haemoglobinopathy. The term SCD actually comprises several different states: sickle cell anaemia, but also compound heterozygous states including sickle cell/haemoglobin C (D) and sickle cell/ -thalassaemia (C).

Do not forget that the haemoglobin molecule consists of four chains, and there are 3 different forms: haemaglobin A ( 2 2), haemoglobin A2 ( 2 2) and haemoglobin F ( 2 2). The proportions of the different forms vary with age – haemoglobin F predominates before birth, but concentrations of haemaglobin A and A2 increase after birth, with haemoglobin A predominating. In sickle-cell anaemia a point mutation in the -globin chain of haemoglobin (found on chromosome 11) results in the hydrophilic amino acid glutamic acid being replaced by the hydrophobic amino acid valine at the 6th position. This promotes aggregation of the haemoglobin chains in conditions of low oxygen, distorting the red blood cells so they adopt a sickle shape. These cells become adherent to the endothelieum of post capillary venules, causing retrograde capillary obstruction which can lead to painful crises.

Sickle cell trait is not the same as sickle cell disease – they have one abnormal allele of the -haemoglobin gene, but are asymptomatic and have no sickle cells on the blood film unlike in this case.
Sickle cell/ -thalassaemia is a variant of sickle cell anaemia, where an individual inherits one haemoglobin gene from a parent who is a car- rier of -thalassemia and and the other from a parent who is a carrier of sickle cell anaemia. The exact -thalassaemia mutation inherited will determine the severity of the disease. A similar situation occurs in sickle cell/haemoglobin C, where the patient has one gene coding for haemo- globin C – these patients may have a mild splenomegaly and haemolytic anaemia. Haemoglobin C is similar to haemoglobin S in that it compris- es two normal alpha chains and two variant beta chains in which lysine has replaced glutamic acid at position 6. Both sickle cell/ -thalassaemia and sickle cell/haemoglobin C are much less likely in this scenario than sickle cell disease.

80
Q
  1. Haematological neoplasms (2)
A Diffuse large B-cell lymphoma
B Burkitt lymphoma
C Follicular lymphoma
D Small lymphocytic leukaemia
E Mantle cell lymphoma
F Peripheral T-cell lymphoma 
G Mycosis fungoides
H Angiocentric lymphoma
I Hodgkin’s lymphoma

1 A 5-year-old boy is seen by a volunteer doctor at an Ethiopian refugee camp. On examination the child has a prominent swelling on the left side of his jaw. A tissue sample of the mass demonstrates a ‘starry sky’ appearance on light microscopy.

A

1) B

Burkitt lymphoma (BL; B) is a haematological cancer of B lymphocytes caused by latent Epstein–Barr viral (EBV) infection and is most prevalent in Africa, affecting children and teenagers. Subtypes of BL include endemic, sporadic and immunodeficiency-associated disease. Endemic BL presents with a mandibular mass whereas non-endemic types present with an abdominal mass. All forms are highly associated with translocations of the c-myc gene on chromosome 8 (the most common with the Ig heavy chain on chromosome 14). A ‘starry sky’ appearance is characteristic when viewing BL cells under microscopy.

Small lymphocytic lymphoma (SLL; D) is indistinguishable from chronic lymphocytic leukaemia (CLL) in terms of genetics and morphology. SLL more commonly presents with greater peripheral blood lymphocytosis than CLL.

Peripheral T-cell lymphomas (F) are a group of heterogeneous mature T-cell lymphomas that are not easily classified. They usually present in adulthood and have an aggressive course.

Mycosis fungoides (G) is a cutaneous T-cell lymphoma most commonly occurring in elderly men. It can present with rash-like lesions that may appear similar to eczema or psoriasis.

Angiocentric lymphoma (H) presents in adulthood as cutaneous masses most commonly in the nasal area. Tumour cells will express NK-cell markers and commonly may be infected with EBV.

81
Q
  1. Haematological neoplasms (2)
A Diffuse large B-cell lymphoma
B Burkitt lymphoma
C Follicular lymphoma
D Small lymphocytic leukaemia
E Mantle cell lymphoma
F Peripheral T-cell lymphoma 
G Mycosis fungoides
H Angiocentric lymphoma
I Hodgkin’s lymphoma

2 A 52-year-old man presents to his GP with painless lymphadenopathy which he describes as having fluctuated in size over the past month, as well as experiencing night sweats and weight loss. He also mentions the lumps become painful when he drinks alcohol. Further biopsy of the lumps reveals the presence of Reed–Sternberg cells.

A

2) I

Hodgkin’s lymphoma (I) results from the proliferation of B cells from the germinal centre. The pathogenesis is linked to EBV infection which activates NF-kappaB, preventing apoptosis of infected cells. Release of IL-5 from B-cells activates eosinophils, prolonging the life of B cells further. Histologically, Hodgkin’s lymphoma is characterized by the presence of Reed–Sternberg cells (binucleate/multinucleate cells with abundant cytoplasm, inclusion-like nucleoli and surrounded by eosinophils). Lymphadenopathy associated with Hodgkin’s lymphoma is usually painless asymmetrical, fluctuates in size and is painful with alcohol intake. Other clinical features include fever, night sweats, weight loss and Pel-Ebstein fever (intermittent fever every 2 weeks). Unlike non-Hodgkin’s lymphoma, extra-nodal involvement is rare.

Small lymphocytic lymphoma (SLL; D) is indistinguishable from chronic lymphocytic leukaemia (CLL) in terms of genetics and morphology. SLL more commonly presents with greater peripheral blood lymphocytosis than CLL.

Peripheral T-cell lymphomas (F) are a group of heterogeneous mature T-cell lymphomas that are not easily classified. They usually present in adulthood and have an aggressive course.

Mycosis fungoides (G) is a cutaneous T-cell lymphoma most commonly occurring in elderly men. It can present with rash-like lesions that may appear similar to eczema or psoriasis.

Angiocentric lymphoma (H) presents in adulthood as cutaneous masses most commonly in the nasal area. Tumour cells will express NK-cell markers and commonly may be infected with EBV.

82
Q
  1. Haematological neoplasms (2)
A Diffuse large B-cell lymphoma
B Burkitt lymphoma
C Follicular lymphoma
D Small lymphocytic leukaemia
E Mantle cell lymphoma
F Peripheral T-cell lymphoma 
G Mycosis fungoides
H Angiocentric lymphoma
I Hodgkin’s lymphoma

3 A 60-year-old man presents to his GP with malaise, night sweats and weight loss. On examination the patient is found to have generalized lymphadenopathy and hepatomegaly. Cytogenetic investigation a few weeks later by a haematologist reveals a translocation between chromosomes 11 and 14, which has caused overexpression of the BCL-2 protein.

A

3) E

Mantle cell lymphoma (MCL; E) is an aggressive B-cell lymphoma primarily affecting elderly men. The most common cause is a translocation between chromosomes 11 and 14, involving the BCL-1 locus and Ig heavy chain locus, therefore leading to over-expression of cyclin D1. Over-expression of cyclin D1 leads to dysregulation of the cell cycle. Clinically, generalized lymphadenopathy, as well as BM and liver infiltration, are common. Hodgkin’s lymphoma can be split into classical and lymphocyte predominant nodular (LPN) subtypes.

Small lymphocytic lymphoma (SLL; D) is indistinguishable from chronic lymphocytic leukaemia (CLL) in terms of genetics and morphology. SLL more commonly presents with greater peripheral blood lymphocytosis than CLL.

Peripheral T-cell lymphomas (F) are a group of heterogeneous mature T-cell lymphomas that are not easily classified. They usually present in adulthood and have an aggressive course.

Mycosis fungoides (G) is a cutaneous T-cell lymphoma most commonly occurring in elderly men. It can present with rash-like lesions that may appear similar to eczema or psoriasis.

Angiocentric lymphoma (H) presents in adulthood as cutaneous masses most commonly in the nasal area. Tumour cells will express NK-cell markers and commonly may be infected with EBV.

83
Q
  1. Haematological neoplasms (2)
A Diffuse large B-cell lymphoma
B Burkitt lymphoma
C Follicular lymphoma
D Small lymphocytic leukaemia
E Mantle cell lymphoma
F Peripheral T-cell lymphoma 
G Mycosis fungoides
H Angiocentric lymphoma
I Hodgkin’s lymphoma

4 A 40-year-old woman is referred to a haematologist after she is found to have generalized, painless lymphadenopathy. A report on tumour cell morphology states the presence of both centrocytes and centroblasts.

A

4) C

Follicular lymphoma (C) is caused most commonly by a translocation between chromosomes 14 and 18, leading to over-expression of the BCL-2 protein. Over-expression of BCL-2 causes inhibition of apoptosis, promoting the survival of tumour cells. Tumour cells in follicular lymphoma are characterized by centrocytes (small B cells with irregular nuclei and reduced cytoplasm) and centroblasts (larger B cells with multiple nuclei). Clinical features include painless, generalized lymphadenopathy. Follicular lymphoma usually presents in middle-aged patients and has a non-aggressive course but is difficult to cure.

Small lymphocytic lymphoma (SLL; D) is indistinguishable from chronic lymphocytic leukaemia (CLL) in terms of genetics and morphology. SLL more commonly presents with greater peripheral blood lymphocytosis than CLL.

Peripheral T-cell lymphomas (F) are a group of heterogeneous mature T-cell lymphomas that are not easily classified. They usually present in adulthood and have an aggressive course.

Mycosis fungoides (G) is a cutaneous T-cell lymphoma most commonly occurring in elderly men. It can present with rash-like lesions that may appear similar to eczema or psoriasis.

Angiocentric lymphoma (H) presents in adulthood as cutaneous masses most commonly in the nasal area. Tumour cells will express NK-cell markers and commonly may be infected with EBV.

84
Q
  1. Haematological neoplasms (2)
A Diffuse large B-cell lymphoma
B Burkitt lymphoma
C Follicular lymphoma
D Small lymphocytic leukaemia
E Mantle cell lymphoma
F Peripheral T-cell lymphoma 
G Mycosis fungoides
H Angiocentric lymphoma
I Hodgkin’s lymphoma

5 A 62-year-old HIV-positive man presents to a haematologist with a 3-month history of weight loss and tiredness. On examination, the patient has a mass on his neck which the patient states has been rapidly growing. Staining of biopsy tissue demonstrates the present of large B cells which are positive for EBV.

A

5) A

Diffuse large B-cell lymphoma (DLBL; A) is a haematological malignancy most commonly affecting the elderly, characterized by large lymphocytes which have a diffuse pattern of growth. Common chromosomal abnormalities which contribute to the development of DLBL include the t(14;18) translocation which is characteristic of follicular lymphoma; this suggests that follicular lymphoma may undergo a degree of transformation to cause DLBL in such circumstances. Tumour cells that have follicular lymphoma morphology may be present at other sites. 2 subtypes of DLBL have been described, both of which are assoc with immunodeficiency: immunodeficiency-associated large B-cell lymphoma (linked to latent EBV infection) and body cavity-based large cell lymphoma (linked to HHV8 infection).

Small lymphocytic lymphoma (SLL; D) is indistinguishable from chronic lymphocytic leukaemia (CLL) in terms of genetics and morphology. SLL more commonly presents with greater peripheral blood lymphocytosis than CLL.

Peripheral T-cell lymphomas (F) are a group of heterogeneous mature T-cell lymphomas that are not easily classified. Usually present in adulthood and have an aggressive course.

Mycosis fungoides (G) is a cutaneous T-cell lymphoma most commonly occurring in elderly men. It can present with rash-like lesions that may appear similar to eczema or psoriasis.

Angiocentric lymphoma (H) presents in adulthood as cutaneous masses most commonly in the nasal area. Tumour cells will express NK-cell markers and commonly may be infected with EBV.

85
Q
  1. Haemoglobinopathies in children (2)
    A 7-year-old child has known sickle cell disease. He presents with a 5-day history of fever, shortness of breath and extreme fatigue. His mother reports that his younger brother, who also has SCD, has been feeling unwell too recently. A blood test for the patient reveals a severe anaemia and low reticulocyte count. He has most likely developed:
A Splenic sequestration
B Pneumococcal infection
C Vaso-occlusive crisis
D Folic acid deficiency
E Parvovirus B19 infection
A

E

Aplastic crises caused by parvovirus B19 infection (E) can occur in patients with SCD. can present with acute worsening of the patient’s baseline anaemia, which might manifest as SOB and fatigue. The fever points to an infectious cause. The virus affects erythropoiesis by invading erythrocyte precursors and destroying them. Infants and children with SCD initially have no immunity to parvovirus B19, and their first exposure can lead to pure red cell aplasia. In a normal individual the virus blocks red cell production for 2 or 3 days with little consequence, but it can be life threatening in sickle cell patients in whom the red cell life span is already shortened. This can lead to profound anaemia over the course of just a few days, and a dramatic drop in the reticulocyte count. Serum IgM antibodies to parvovirus B19 can confirm the diagnosis, and blood transfusion may be required.

Splenic sequestration (A) is a potentially fatal emergency caused by the acute pooling of a large percentage of circulating red cells in the spleen when it is enlarged. It would not present in the way described in this case, but with an abdomen that can become bloated and hard with signs of circulatory failure. This is less common in older children and adults because recurrent infarction has often left the spleen small and fibrotic, but is a possibility in younger children.

Sickle cells can also become adherent to the endothelieum of post capillary venules, causing retrograde capillary obstruction which can lead to painful vaso-occlusive crises (C). Pain would be a more significant feature of the presentation than in this case, and a severe anaemia and reticulocytopenia would not normally occur as a consequence.

Patients with SCD are at high risk of pneumococcal infection (B) after a splenectomy, and this would not typically cause a low reticulocyte count.

Folic acid deficiency (D) is more common in sickle cell patients due to hyperplastic erythropoesis, and is a particular problem in children who require folic acid for growth spurts. This is less likely to present as acutely as in this case, and the fact that the patient’s sibling is also affected makes an infectious trigger more plausible.

86
Q
  1. Anaemia (1)

A 26-year-old pregnant woman is found to have an Hb of 9.5g/dL on a routine blood test, with an MCV of 70. Serum electrophoresis reveals an Hb A2 of 3.9% and Hb A of 96.1%. Her ferritin levels are normal. The most likely diagnosis is:

A Iron deficiency anaemia
B Cooley’s anaemia
C  beta-Thalassaemia intermedia
D  beta-Thalassaemia minor
E  alpha-Thalassaemia
A

Beta-Thalassaemias = group of genetic haemoglobinopathies that result in reduced/absent formation of Hb beta chains leading to anaemia of varying severity. Prevalent in Middle East, Central, South and South East Asia, Southern China and around the Mediterranean. There are 3 main forms: thalassaemia major, thalassaemia intermedia and thalassaemia minor.

In Beta-thalassaemia minor (D) only one of the -globulin alleles is mutated, so these individuals usually only have a well-tolerated microcytic anaemia (Hb >9g/dL) which is clinically asymptomatic. Picked up on a routine blood test, with a low MCH and significantly low MCV (<80fL). They also have an increase in the fraction of Hb A2, as in this case. In most people the fraction of Hb A2 ( 2 2) will be 1.5–3.5%, but in Beta-thalassaemia minor the proportion of Hb A2 is >3.5–4% to compensate for the reduced amount of normal haemoglobin, and they might have a slight increase in HbF. It can worsen in pregnancy, as in this case.

Beta -Thalassaemia intermedia (C) is a condition that lies in between the minor and major forms. moderate anaemia and sometimes splenomegaly. Do not require blood transfusions.

Cooley’s anaemia (B), or Beta-thalassaemia major, is the homozygous form. These patients would present much earlier than in this case, usually in the first year of life with failure to thrive and a severe microcytic anaemia. Hepatosplenomegaly and bossing of the skull may occur due to extramedullary haemopoesis (i.e. red blood cells being produced outside the BM). Tx: lifelong blood transfusions with iron chelators to prevent overload, splenectomy if hypersplenism persists and BMT can offer the chance of a cure.

Alpha-Thalassaemia (E) affects genes coding for alpha-Hb chains on chromosome 16. There are varying forms which would not cause the increase in Hb A2.
If 1 of the 4 alpha-haemoglobin genes is deleted, the patient is clinically normal. If 2 are deleted, the patient has a low MCV but is asymptomatic.
If 3 are deleted the disease is called HbH. might have features of haemolysis (hepatosplenomegaly & jaundice), anaemia of moderate severity.
If all 4 genes are deleted = ‘Bart’s hydrops’, and death occurs in utero.

IDA (A) can cause a low Hb and MCV as in this case, but the ferritin would usually be reduced.